Контрольная работа по физике ответы: ГДЗ по физике 7 класс контрольные и самостоятельные работы Громцева

Содержание

Итоговая контрольная работа по физике 7 класс. 2 варианта с ответами.

Фамилия, имя _______________________________________________

Задание в тестовой форме за год 7 класс.

Вариант 1.

При выполнении заданий с выбором ответа обведите кружком номер правильного ответа в экзаменационной работе.

  1. Установите соответствие:

I.

Стальной шарик —

1.

физическое тело

II.

Диффузия —

2.

физическая величина

III.

Молекула —

3.

физическое явление

IV.

Расстояние —

V.

Объем —


Ответ: I.___ II.___ III.___ IV.___ V.___

  1. Укажите название приборов, предназначенных для измерения следующих физических величин.

I.

Масса

1.

Мензурка.

II.

Сила

2.

Секундомер

III.

Скорость

3.

Термометр.

IV.

Температура

4.

Динамометр.

V.

Объем тел неправильной формы

5.

Рычажные весы.

VI.

Давление

6.

Спидометр

7.

Барометр

Ответ: I.___ II.___ III.___ IV.___ V.___VI.____

  1. Установить соответствие.

Формула

1.

F

А)

abc

2.

ρ

Б)

s/t

3.

υ

В)

mg

4.

v

Г)

m/v

5.

р

Д)

mv

Е)

F/S

Ответ: 1. ____ 2. ____ 3. ____ 4. ____5._____

  1. Плотность цинка равна 7100 кг/м3. Чему равна плотность цинка в г/см3?

А). 7,1 г/см3 Б). 7100000 г/см3 В). 0,0071 г/см3 Г). 0

  1. Укажите единицы измерения следующих физических величин:

I.

Масса

1.

м3

II.

Сила

2.

Н

III.

Плотность

3.

кг

IV.

Объем

4.

кг/м3

V.

Скорость

5.

м/с

VI.

Площадь

6.

м

VII

Давление

7.

м2

8.

Па

Ответ: I.___ II.___ III.___ IV.___ V.___ VI.___ VII._____

6. На рисунке дан график пути равномерного движения. Определите по графику скорость движения.

А) 10 м/с

Б) 20 м/с

В) 30 м/с

Г) 40 м/с

7. Чему равна цена деления измерительного цилиндра, изображенного на рисунке?


8. Чему равна равнодействующая трех сил, приложенных к телу в точке А? Куда она направлена?


А) 2Н, вправо Б) 1Н, влево В) 5Н, вправо Г) 3Н, влево

9. Кусок металла массой 461,5г имеет объем 65 см3. Чему равна плотность вещества?

А) 461,5 г/см3 Б) 65 г/см3 В) 7,1 г/см3 Г) ≈0.1 г/см3

10. Автомобиль движется равномерно. Используя рисунок, определите силу тяги, действующую на автомобиль.


А) 100Н Б) 600Н В) 500Н Г) 0


11. В каком состоянии находится вещество, если оно сохраняет свой объем и форму?

А) в жидком; Б) в твердом; В) в газообразном;

Г) или в жидком или в газообразном.

12. Как называется явление, при котором соприкасающиеся вещества сами собой смешиваются друг с другом?

А) кипение; Б) диффузия; В) таяние; Г) плавление.

13. На рисунке изображены три сосуда с водой. Площади дна сосудов равны. Сравните давления

p1 , p2 и p3 жидкости на дно сосуда


А) p1= p2= p3

Б) p1< p2<p3

В) p1= p2< p3

Г) p1= p2> p3


14. Рычаг находится в равновесии, если отношение плеч сил

F1 и F2 равно:

А) 0 Б) 1 В) 2:5 Г) 5:2

15. Потенциальная энергия какого из поднятых над землей камней больше— в случае а? в случае б?


1) 1

2) 2

3) одинакова

4) недостаточно данных для ответа

Ответ: а ____________;

б _______________

16. В каком положении брусок производит наибольшее давление?

А) в первом Б) во втором

В) в третьем Г) во всех одинаково

17. Чему примерно равна Архимедова сила, действующая на тело объемом 2 м3, полностью погруженное в жидкость плотностью 1000 кг/м3?

А)2000Н Б) 5000 Н В) 10000 Н Г) 20000 Н

18. Шприцом набирают воду из стакана. Почему вода поднимается вслед за поршнем?

А) Молекулы воды притягиваются молекулами поршня

Б) Поршень своим движением увлекает воду

В) При подъеме между поршнем и водой образуется безвоздушное пространство, куда под давлением наружного воздуха устремляется вода

Г) Среди приведенных объяснений нет правильного

Фамилия, имя _______________________________________________

Задание в тестовой форме за год 7 класс.

Вариант 2.

При выполнении заданий с выбором ответа обведите кружком номер правильного ответа в экзаменационной работе.

  1. Укажите единицы измерения следующих физических величин:

I.

Масса

1.

м3

II.

Сила

2.

Па

III.

Плотность

3.

кг

IV.

Объем

4.

кг/м3

V.

Скорость

5.

м/с

VI.

Площадь

6.

м

VII.

Давление

7.

м2

8.

Н

Ответ: I.___ II.___ III.___ IV.___ V.___ VI.___VII._______

  1. Тело движется равномерно и прямолинейно со скоростью 54 км/ч. Чему равна скорость тела, выраженная в м/с?

А) 15 м/с Б) 54 м/с В) 54000 м/с Г) 0,015 м/с.

3. Составьте простые предложения.

I.

Скорость

1.

физическое тело

II.

Движение

2.

физическая величина

III.

Тележка

3.

физическое явление

IV.

Масса

V.

Путь

Ответ: I.___ II.___ III.___ IV.___ V.___

  1. На рисунке дан график пути равномерного движения. Определите по графику скорость движения


А) 350 км/ч Б) 6 км/ч В) 50 км/ч Г) 50 м/с

5. Укажите название приборов, предназначенных для измерения следующих физических величин.

I.

Мензурка.

1.

Масса

II.

Спидометр.

2.

Сила

III.

Термометр.

3.

Скорость

IV.

Динамометр.

4.

Температура

V.

Рычажные весы.

5.

Объем тел неправильной формы

VI.

Барометр

6.

Давление

7.

Время

Ответ: I.___ II.___ III.___ IV.___ V.___ VI.________

6.Установить соответствие.

Формула

1.

F

А)

abc

2.

ρ

Б)

s/t

3.

υ

В)

mg

4.

v

Г)

m/v

5.

р

Д)

mv

Е)

F/S

Ответ: 1.___ 2.___ 3.___ 4.____5._____

7. Чему равна цена деления измерительного цилиндра, изображенного на рисунке?

8. Чему равна равнодействующая трех сил, приложенных к телу в точке А? Куда она направлена?


А) 2Н, влево Б) 1Н, влево В) 5Н, вправо Г) 3Н, вправо

9. В аквариум вместимостью 15 м3 налита вода. Какова масса воды в аквариуме? Плотность воды примите равной 1000 кг/м3.

А) 15000 кг Б) 0,015 кг В) 15 кг Г) 1000 кг

10. Используя рисунок, определите силу тяги, действующую на лодку.

Масштаб: 1 деление =1,5 Н

А) 4 Н Б) 6 Н В) 1,5 Н Г) 40 Н

11. В каком состоянии находится вещество, если оно сохраняет объем, но легко меняет форму?

А) в твердом; Б) В жидком; В) в газообразном;

Г) или в жидком или в газообразном.

12. В каких телах диффузия при одинаковых температурах происходит быстрее?

А) в газах; Б) в жидкостях;

В) в твердых телах; Г) во всех одинаково.

13. На рисунке изображены три сосуда с водой. Площади дна сосудов равны. Сравните давления p1 , p2 и p3 жидкости на дно сосуда

А) p1< p2<p3

Б) p1= p2< p3

В) p1= p2= p3

Г) p1= p2> p3

14. Рычаг находится в равновесии, если отношение сил F1 и F2 равно:

А) 0 Б) 1 В) 2:5 Г) 5:2

15. Какой из грузовиков обладает большей кинетической энергией в случае а? в случае б?

1) 1

2) 2

3) одинакова

4) недостаточно данных для ответа

Ответ: а _____________;

б ________________

16. Чему примерно равна Архимедова сила, действующая на тело объемом 2 м3, наполовину погруженное в жидкость плотностью 1000 кг/м3?

А)2000Н Б) 5000 Н В) 10000 Н Г) 20000 Н

17. Если сила тяжести, действующая на погруженное в жидкость тело, больше архимедовой силы, то тело…

А). Всплывает В). Находится в равновесии в любом месте жидкости

Б). Тонет Г). Плавает на поверхности жидкости

18. В каком положении брусок производит наименьшее давление?

А) в первом Б) во втором

В) в третьем Г) во всех одинаково

Вариант 1

1

2

3

4

5

6

7

8

9

10

11

12

13

14

15

16

17

18

№ правильного ответа

I –1

II –3

III –1

IV –2

V -2

I –5

II –4

III –6

IV –3

V –1

VI -7

1 –В

2 –Г

3 –Б

4 –А

5 -Е

А

I –3

II –2

III –4

IV –1

V –5

VI –7

VII -8

А

В

В

В

Б

Б

Б

А

В

а -1

б — 1

А

Г

В

Вариант 2

1

2

3

4

5

6

7

8

9

10

11

12

13

14

15

16

17

18

№ правильного ответа

I –3

II –8

III –4

IV –1

V –5

VI –7

VII -2

А

I –2

II –3

III –1

IV –2

V — 2

В

I –5

II –3

III –4

IV –2

V –1

VI -6

1 –В

2 –Г

3 –Б

4 –А

5 -Е

Б

В

А

Б

Б

А

Г

а — 1

б — 2

В

Б

В

Вариант 1

1

2

3

4

5

6

7

8

9

10

11

12

13

14

15

16

17

18

Баллы

5

6

5

1

7

1

1

1

1

1

1

1

1

1

2

1

1

1

Вариант 2

1

2

3

4

5

6

7

8

9

10

11

12

13

14

15

16

17

18

Баллы

7

1

5

1

6

5

1

1

1

1

1

1

1

1

2

1

1

1

Критерии оценивания:

Меньше 18 баллов – 2

19-27 баллов – 3

28 – 35 баллов – 4

36-38 баллов — 5

Контрольная работа по физике 7 класс «Силы вокруг нас»

Контрольная работа по физике по теме

«Силы вокруг нас»

Пояснительная записка

Работа состоит из 15 заданий, которые разделены на три части.

Часть 1 содержит 8 заданий с выбором ответа. К каждому заданию дается несколько вариантов ответов, из которых правильный только один. За каждый верный ответ ставится один балл.

Часть 2 содержит 2 задание на установление соответствия. Правильный ответ оценивается в два балла.

В части 3 предлагается поработать с текстом и вставить пропущенные слова, подходящие по смыслу и задачи для контроля практических умений и навыков учащихся по решению стандартных задач, на которую следует дать полное решение, включающее запись краткого условия задачи, запись формул, применение которых необходимо и достаточно для решения задачи, и ответ в числовом виде. Правильное решение оценивается до трех баллов, в зависимости от полноты и правильности ответа.

Оценка знаний учащихся по итогам выполнения теста производится в соответствии с таблицей:

Оценка

«2»

«3»

«4»

«5»

Баллы

16 и ниже

14-17

18-24

25-27

I вариант

Часть 1

1. Сила не может являться причиной изменения:

А. массы тела

Б. плотности тела

В. формы тела

Г. скорости тела

2. Силу, возникающую при движении одного тела по поверхности другого и направленную против движения, называют:

А. силой тяжести.

Б. весом тела

В. силой упругости

Г. силой трения

3. Сила тяжести -это сила, которая возникает вследствие взаимодействия Земли и….

А. Луны

Б. тела на поверхности Земли

В. планет Солнечной системы

Г. Солнца

4. В соревновании по перетягиванию каната участвуют три человека. Один, прикладывая силу 550 Н, тянет канат вправо, а двое с силами 200 Н и 250 Н- влево. Какова равнодействующая этих сил? В каком направлении будет перемещаться канат?

А.  550 Н, вправо. 

Б. 450 Н, влево. 

В. 100 Н, влево. 

Г. 100 Н, вправо

 5. Определите, на каком из рисунков правильно изображена сила тяжести, действующая на тело:


А. Рис.1

Б. Рис. 2

В. Рис.3

Г. Рис.4

6. Какое из утверждений верно?

а) Сила является мерой взаимодействия тел.

б) Результат действия силы не зависит от точки приложения силы

А. только а.

Б. только б

В. оба верны

Г. оба не верны

7. К видам трения не относится

А. трение покоя

Б. трение движения

В. трение скольжения

Г. трение качения

8. Сила, равная 10 Н, растягивает пружину на 4 см. Коэффициент упругости пружины равен:

А. 2,5 Н/м

Б. 40 Н/м

В. 25 Н/м

Г. 250 Н/м

Часть 2

9.Установите соответствие между физическими величинами и формулами их определения. К каждой позиции первого столбца подберите соответствующую позицию второго и запишите в таблицу выбранные цифры под соответствующими буквами.

А. Сила тяжести                                          1. mg

Б. Вес тела                                                 2. k ∆l

В. Сила упругости                                       3. ρm

                                                                  4. vS

10. Установите соответствие между картинками и видами деформаций, которые испытывает позвоночник человека в следующих ситуациях

1. Деформация сдвига

2. Деформация изгиба

3. Деформация кручения

4. Деформация растяжения

5. Деформация сжатия

Часть 3.

11. Вставьте в текст пропущенные слова.

Сила – это причина изменения________________ движения тела. Различают несколько видов сил различной природы. Сила, с которой Земля притягивает к себе все тела, называется _________ ____________. Сила упругости возникает при ________________ тела. Вес тела следует отличать от силы _______________. Сила _______________ возникает вследствие взаимодействия тела с ______________, а _____________ — в результате взаимодействия тела с опорой или ____________. При движении одного тела по ________________ другого возникает сила ______________.

12. Определите вес железнодорожной платформы, массой 24 тонны   Ускорение свободного падения принять равной 10 Н/кг.

13. Мопед «Рига-16» весит 490 Н. Какова его масса? Ускорение свободного падения 9,8 Н/кг.

14. В игре по перетягиванию каната участвуют четыре человека. Два из них тянут канат в одну сторону (влево) с силой 120Н и 280Н, два – в другую сторону (вправо) с силой 100 Н и 250Н. В каком направлении будет двигаться канат и чему равна равнодействующая сила?

15.  Найдите вес 25 л. керосина. Ускорение свободного падения 9,8 Н/кг.

Ответы

Часть 1

1

2

3

4

5

6

7

8

Б

Г

Б

Г

В

А

Б

Г

Часть 2

9.

10.

Часть 3

11. Сила – это причина изменения скорости движения тела. Различают несколько видов сил различной природы. Сила, с которой Земля притягивает к себе все тела, называется силой тяжести. Сила упругости возникает при деформации тела. Вес тела следует отличать от силы тяжести. Сила тяжести возникает вследствие взаимодействия тела с Землей, а вес — в результате взаимодействия тела с опорой или подвесом. При движении одного тела по поверхности другого возникает сила трения.

12. 240 000Н

13. 50 кг

14. 50 Н, вправо

15. 196 Н

Диагностическая итоговая контрольная работа по физике в 7 классе

Муниципальное бюджетное общеобразовательное учреждение «Колонтаевская средняя общеобразовательная школа»

Льговского района Курской области

ДИАГНОСТИЧЕСКАЯ РАБОТА

ФИЗИКА

7 класс

Подготовила:

Сикачёва И. В., учитель физики

2019 год

Описание контрольных измерительных материалов для осуществления оценки качества образования по физике обучающихся 7 классов

  1. Назначение работы

КИМ разработаны в соответствии с письмом Федеральной службы по надзору в сфере образования и науки (Рособнадзор) №05-71 от 16.03.2018 «О направлении рекомендаций по повышению объективности оценки образовательных результатов».

Назначение диагностической работы по учебному предмету «ФИЗИКА» – оценить уровень общеобразовательной подготовки учащихся 7 классов по темам «Физические явления. Первоначальные сведения о строении вещества. Движение и взаимодействие тел. Давление твердых тел, жидкостей и газов. Работа и мощность.»

Осуществить оценку предметных и метапредметных компетенций, в том числе овладение межпредметными понятиями и способность использования универсальных учебных действий (УУД) в учебной и познавательной практике.

Результаты могут быть использованы общеобразовательными организациями, муниципальными региональными органам исполнительной власти, осуществляющими государственное управление в сфере образования, для анализа текущего состояния муниципальных и региональных систем образования и формирования программ их развития.

2. Документы, определяющие содержание работы

Содержание и структура диагностической работы определяются на основе Федерального государственного образовательного стандарта основного общего образования по физике (Приказ Министерства образования и науки РФ от 17 декабря 2010 г. № 1897 «Об утверждении федерального государственного образовательного стандарта основного общего образования») с учётом Примерной основной образовательной программы основного общего образования (одобрена решением федерального учебно-методического объединения по общему образованию (протокол от 08.04.2015 № 1/15).

Задания КИМ не ориентированы на какую-либо конкретную линию учебников, их содержание соответствует всем учебникам, включённым в Федеральный перечень Минобрнауки РФ на 2018–2019 учебный год

3. Структура работы

Каждый вариант диагностической работы состоит из двух частей и включает 13 заданий, различающихся формой и уровнем сложности.

Часть 1 содержит 10 заданий с выбором ответа. К каждому заданию приводится 3 варианта ответа, из которых верен только один.

Часть 2 включает 3 задания, к которым требуется привести краткий ответ в виде набора цифр или числа. Задания 11 и 12 представляют собой задания на установление соответствия позиций, представленных в двух множествах. Задание 13 содержит расчетную задачу, для которой необходимо привести полное решение.

4. Кодификаторы проверяемых элементов содержания и требований к уровню подготовки учащихся

В таблице1приведен кодификатор проверяемых элементов содержания

Таблица1

Код

раздела

Код элемента

Проверяемые элементы содержания

1

Тепловые явления. Первоначальные сведения о строении вещества.

1.1

Строение вещества. Молекулы.

1.2

Тепловое движение атомов и молекул. Связь температуры веще­ства со скоростью хаотического движения частиц. Диффузия

1.3

Три состояния вещества. Модели строения газов, жидкостей и твердых тел.

2

Механические явления.

2.1.

Механическое движение. Траектория. Путь. Относительность движения.

2.2

Прямолинейное равномерное движение. Скорость.

2. 3

Масса тела. Единицы массы.

2.4

Плотность вещества.

2.5

Взаимодействие тел. Сила.

2.6

Явление тяготения. Сила тяжести. Вес тела.

2.7

Сила упругости.

2.8

Сила трения.

2.9

Давление. Единицы давления

2.10

Давление жидкости и газа.

2.11

Атмосферное давление.

2. 12

Действие жидкости и газа на погруженное в них тело. Сила Архимеда.

3

Физика и физические методы изучения природы.

3.1

Физические величины и их измерение.

3.2

Физические приборы.

В таблице 2 приведены требования к уровню подготовки участников

Таблица2

Код

Код контролируемого требования (умения)

Проверяемые требования к уровню подготовки

1

Метапредметные

1. 1

выделять главные, существенные признаки понятий. Строить логическое рассуждение и делать выводы;

1.2

понимать различия между исходными фактами для их объяснения. Строить логическое рассуждение и делать выводы;

1.3

классифицировать информацию по заданным признакам;

1.4

воспринимать, перерабатывать информацию, представленную на рисунке, для ответа на вопрос задания;

1.5

понимать различия между исходными фактами и гипотезами для их объяснения, теоретическими моделями и реальными объектами, овладение УУД на примерах гипотез для объяснения известных фактов и экспериментальной проверки выдвигаемых гипотез, разработки теоретических моделей процессов или явлений;

1. 6

освоение приёмов действий в нестандартных ситуациях, овладение эвристическими методами решения проблем.

2

Предметные:

2.1

распознавать, понимать и объяснять методы исследования;

2.2

распознавать и понимать различия между понятиями

«физическая величина» и «единица измерения»;

2.3

понимать смысл изученных физических понятий и явлений (узнавать описание явления, его определение, различать условия протекания явления, объяснять явления на основе имеющихся знаний).

2. 4

понимать различие между броуновским движением, диффузией;

2.5

знать и понимать смысл физических законов (узнавать словесную формулировку закона, применять закон для анализа процессов на качественном и расчетном уровнях, различать проявления законов в окружающей жизни и их использование для создания технических устройств).

2.6

понимать различие основных признаков моделей строения газов, жидкостей и твёрдых тел;

2.7

владение основами методологических знаний и умений (различать/формулировать цели проведения, порядок проведения и выводы описанных наблюдений и опытов, определять цену деления, пределы измерения прибора и записывать его показания).

2.8

объяснять на основе имеющихся знаний о строении веществе а большую сжимаемость газов, малую сжимаемость жидкостей и твёрдых тел; объяснять различия основных признаков моделей строения газов, жидкостей и твёрдых тел;

2.9

применять разнообразные способы выполнения расчётов для нахождения неизвестной величины в соответствии с условиями поставленной задачи.

5. Распределение заданий по позициям кодификатора

Код

Контролируемые виды деятельности

Код требований к уровню подготовки выпускников

Коды проверяемых элементов содержания

Уровень сложности задания

1

выделять главные, существенные признаки понятий. Строить логическое рассуждение и делать выводы;

1.1

1.1-2.9

Б

понимать различия между исходными фактами для их объяснения. Строить логическое рассуждение и делать выводы;

1.2

2.2, 2.4,2.12

Б

классифицировать информацию по заданным признакам;

1.3

2.9

Б

воспринимать, перерабатывать информацию, представленную на рисунке, для ответа на вопрос задания;

1.4

2. 9

Б

понимать различия между исходными фактами и гипотезами для их объяснения, теоретическими моделями и реальными объектами, овладение УУД на примерах гипотез для объяснения известных фактов и экспериментальной проверки выдвигаемых гипотез, разработки теоретических моделей процессов или явлений;

1.5

1.3,

Б

освоение приёмов действий в нестандартных ситуациях, овладение эвристическими методами решения проблем;

1.6

2.5-2.9

Б

2

распознавать, понимать и объяснять методы исследования;

2. 1

2.1,2.2

Б

распознавать и понимать различия между понятиями

«физическая величина» и «единица измерения»;

2.2

3.1, 3.2

Б

понимать смысл изученных физических понятий и явлений (узнавать описание явления, его определение, различать условия протекания явления, объяснять явления на основе имеющихся знаний).

2.3

2.9, 2.12

Б

понимать различие между броуновским движением, диффузией;

2.4

1. 2

Б

знать и понимать смысл физических законов (узнавать словесную формулировку закона, применять закон для анализа процессов на качественном и расчетном уровнях, различать проявления законов в окружающей жизни и их использование для создания технических устройств).

2.5

2.12

Б

понимать различие основных признаков моделей строения газов, жидкостей и твёрдых тел;

2.6

1.1

Б

владение основами методологических знаний и умений;

2.7

3. 1, 3.2

П

объяснять на основе имеющихся знаний о строении веществе а большую сжимаемость газов, малую сжимаемость жидкостей и твёрдых тел; объяснять различия основных признаков моделей строения газов, жидкостей и твёрдых тел;

2.8

1.1-1.3

Б

применять разнообразные способы выполнения расчётов для нахождения неизвестной величины в соответствии с условиями поставленной задачи.

2.9

2.2,2.3

П

6. Распределение заданий диагностической работы по уровню сложности

В таблице 3 представлена информация о распределении заданий диагностической работы по уровню сложности.

Таблица3

Уровень сложности

Количество заданий

Максимальный первичный балл

Процент (%) от максимального первичного балла

1

Базовый

10

10

62

2

Повышенный

3

6

38

Итого

13

16

100

6. Распределение заданий диагностической работы по разделам (модулям) и уровню сложности

В таблице 4 представлена информация о распределении заданий диагностической работы по разделам и уровню сложности.

Таблица 4

Раздел (модуль)

Кол-во

заданий

Баллы

Уровни сложности

(количество заданий)

Базовый

Повышенный

1

Тепловые явления. Первоначальные сведения о строении вещества.

1

1

1

2

Механические явления.

10

11

9

1

3

Физика и физические методы изучения природы.

2

4

2

Итого:

13

16

10

3

7. Типы заданий, сценарии выполнения заданий

8. Система оценивания выполнения отдельных заданий и работы в целом

Правильно выполненная работа оценивается 16 баллами.

Правильный ответ на задание 1 – 10 оценивается в 1 балл

Задания 11-13 оцениваются в 2 балла, если нет ошибок, в 1 балл, если допущена одна ошибка или отсутствует один символ при верно указанных других символах, и в 0 баллов, если допущены две ошибки.


Рекомендации по переводу первичных баллов в отметки по пятибалльной шкале

Таблица5

Отметка по пятибалльной шкале

«2»

«3»

«4»

«5»

Первичные баллы

0-5

6-8

8-12

13-16

9. Время выполнения работы

На выполнение всей работы отводится 45 минут.

10. Дополнительные материалы и оборудование

Учащиеся могут пользоваться непрограммируемыми калькуляторами и справочными сведениями

11. Рекомендации по подготовке к стартовой диагностической работе

При подготовке к диагностической работе рекомендуется использовать учебник Физика. 7 кл.: учеб. для общеобразоват. учреждений / А. В. Перышкин. – М.: Дрофа, 2015, Электронное приложение к учебнику, Физика. Сборник вопросов и задач. 7—9 классы (авторы А. Е. Марон, Е. А. Марон, С. В. Позойский)

Диагностическая работа

по ФИЗИКЕ

7 класс

Вариант 1

Инструкция по выполнению работы

На выполнение диагностической работы по физике отводится 45 минут. Работа состоит из двух частей и включает в себя 13 заданий.

Часть 1 содержит 10 заданий с выбором ответа. К каждому заданию приводится 4 варианта ответа, из которых вам нужно выбрать только один.

Часть 2 включает 3 задания. В заданиях 11-12 требуется установить соответствия позиций, представленных в двух множествах и привести краткий ответ в виде набора цифр, которые необходимо записать в указанной таблице. В задании 13 требуется решить задачу на отдельном листе.

При выполнении заданий Вы можете пользоваться черновиком. Обращаем Ваше внимание на то, что записи в черновике не будут учитываться при оценивании работы.

Советуем выполнять задания в том порядке ,в котором они даны. В целях экономии времени пропускайте задания, которые не удается выполнить сразу, и переходите к следующему. Если после выполнения всей работы у Вас останется время, то Вы сможете вернуться к пропущенным заданиям. Постарайтесь выполнить как можно больше заданий.

Желаем успеха!

Часть 1

К каждому из заданий 1-10 даны 4 варианта ответа, из которых только один правильный. Номер этого ответа запишите в поле ответа.

  1. Что из перечисленного не относится к физическим величинам?

А. скорость Б. масса В. Свет

Ответ:

  1. За 5 ч 30 мин велосипедист проехал 99 км. С какой средней скоростью он двигался?

А. 18 км/ч Б. 18м/с В. 20км/мин

Ответ:

  1. Стальной, латунный и чугунный шарики имеют одинаковые объёмы. Какой из них имеет большую массу?

А. Стальной. Б. Чугунный В. Латунный.

Ответ:

  1. Чему равна сила, действующая на тело массой 50 кг, находящееся на поверхности Земли?

А. 0,2 Н Б. 5 Н В. 500 Н

Ответ:

  1. На тело действуют три силы: направленная вверх сила в 10 Н и направленные вниз силы в 9 Н и 5 Н. Куда направлена и чему равна равнодействующая этих сил?

А. направлена вверх и равна 4 Н Б. направлена вниз и равна 4 Н В. Направлена вниз и равна 24 Н

Ответ:

  1. Какое давление оказывает на пол ковер весом 200 Н площадью 4 м³?

А. 50Па Б. 80Па В. 0, 5 Па

Ответ:

  1. Какие две физические величины имеют одинаковые единицы измерения?

А. Сила и работа Б. Работа и мощность В. Работа и энергия

Ответ:

  1. Какова кинетическая энергия самолета массой 20 т, летящего на высоте 10 км со скоростью 150 м/с?

А. 2,25 МДж Б. 225 МДж В. 425 МДж

Ответ:

  1. Если полезная работа при перемещении груза равна 300 Дж, а затраченная при этом работа составила 400Дж, то КПД такого механизма равен:

А. 50% Б. 75% В. 25%

Ответ:

  1. В жидкости находятся два шара одинаковой массы, сделанные из алюминия и меди. На какой из шаров действует самая большая выталкивающая сила?

А. медный Б. Алюминиевый В. Сила одинаковая

Ответ:

Часть 2.

При выполнении заданий 11-12 установите соответствие между содержанием первого и второго столбцов. Для этого каждому элементу первого столбца подберите позицию из второго столбца. Впишите в таблицу внизу задания цифры – номера выбранных ответов.

11) Установите соответствие между физическими законами и учеными, открывшими их. К каждой позиции первого столбца выберите соответствующую цифру из второго столбца и запишите в таблицу выбранные цифры под соответствующей буквой.

А. Открытие явления свободного падения 1). И. Ньютон

Б. Открытие закона всемирного тяготения 2). Б. Паскаль

В. Открытие закона о передаче давления жидкостями 3). А. Эйнштейн

и газами 4). Г. Галилей

5). Р. Броун

12) Установите соответствие между физическими величинами и приборами для их измерения.

К каждой позиции первого столбца выберите соответствующую цифру из второго столбца и запишите в таблицу выбранные цифры под соответствующей буквой.

А. Давление внутри жидкости 1). Барометр

Б. Объем жидкости 2). Манометр

В. Масса 3) Спидометр

4) Весы

5) Мензурка

При выполнении задания 13 решение запишите на отдельном бланке. Сначала укажите номер задания, а затем запишите его решение и ответ. Пишите чётко и разборчиво

13) Какой путь может проехать автомобиль после заправки горючим, если на 100 км пути его движения расходуется 10 кг бензина, а объем топливного бака равен 60 л. Плотность бензина 710 кг/м3?

Диагностическая работа

по ФИЗИКЕ

7 класс

Вариант 2

Инструкция по выполнению работы

На выполнение диагностической работы по физике отводится 45 минут. Работа состоит из двух частей и включает в себя 13 заданий.

Часть 1 содержит 10 заданий с выбором ответа. К каждому заданию приводится 4 варианта ответа, из которых вам нужно выбрать только один.

Часть 2 включает 3 задания. В заданиях 11-13 требуется установить соответствия позиций, представленных в двух множествах и привести краткий ответ в виде набора цифр, которые необходимо записать в указанной таблице. В задании 13 требуется решить задачу на отдельном листе.

При выполнении заданий Вы можете пользоваться черновиком. Обращаем Ваше внимание на то, что записи в черновике не будут учитываться при оценивании работы.

Советуем выполнять задания в том порядке ,в котором они даны. В целях экономии времени пропускайте задания, которые не удается выполнить сразу, и переходите к следующему. Если после выполнения всей работы у Вас останется время, то Вы сможете вернуться к пропущенным заданиям. Постарайтесь выполнить как можно больше заданий.

Желаем успеха!

Часть 1

К каждому из заданий 1-10 даны 4 варианта ответа, из которых только один правильный. Номер этого ответа обведите кружком.

  1. Что из перечисленного не относится к понятию «физическое тело»?

А. кусок мела Б. вода В. Тетрадь

Ответ:

  1. Скорость зайца равна 15 м/с, а скорость догоняющей его лисы равна 72 км/ч. Догонит ли лиса зайца?

А. Да, т.к. скорость у лисы больше Б. Нет, т.к. скорость у лисы меньше В. Нет, т.к. скорости у них равны и расстояние между лисой и зайцем не меняются

Ответ:

  1. Плотность человеческого тела 1070 кг/ м³. Вычислить объём тела человека массой 53, 5 кг.

А. 20 м³; Б. 0, 05 м³; В. 2 м3

Ответ:

  1. С помощью динамометра равномерно перемещают брусок. Чему равна сила трения скольжения между бруском и столом, если динамометр показывает 0,5 Н?

А. 0 Б. 0,5 Н В. 1 Н

Ответ:

  1. В соревнованиях по перетягиванию каната участвуют 4 человека, двое из них тянут канат вправо, прикладывая силы F1 = 250 Н и F2 = 200 Н, двое других тянут влево с силой F3 = 350 Н и F4 = 50 Н. Какова равнодействующая сила?

А. 850 Н Б. 450 Н В. 50 Н

Ответ:

  1. Вычислите силу, действующую на парус яхты площадью 50 м2 при давлении ветра на парус в 100 Па.

А. 5 кН Б. 2 Н В. 500Па

Ответ:

  1. Камень сорвался с горы и падает вниз. Как при падении меняются его импульс и кинетическая энергия?

А. Увеличиваются Б. Уменьшаются В. Кинетическая энергия увеличивается, а импульс уменьшается

Ответ:

  1. Ведро воды из колодца глубиной 3м мальчик поднял первый раз за 20с, а второй раз – за 30с . Одинаковую ли мощность он при этом развивал?

А. Одинаковую Б. Разную; в первый раз мощность была меньше

В. Разную, в первый раз мощность была больше

Ответ:

  1. В трех сосудах различной формы налита вода, высота уровня воды одинакова. В каком из трех сосудов давление на дно наименьшее?

А. Во всех сосудах одинаковое Б. В сосуде 1 В. В сосуде 2 Г . В сосуде 3

Ответ:

  1. Кусок парафина не утонет в

А. воде Б. керосине В. Нефти

Ответ:

ЧАСТЬ 2.

При выполнении заданий 11-12 установите соответствие между содержанием первого и второго столбцов. Для этого каждому элементу первого столбца подберите позицию из второго столбца. Впишите в таблицу внизу задания цифры – номера выбранных ответов.

11). Установите соответствие между физическими величинами и единицами их измерения.

К каждой позиции первого столбца выберите соответствующую цифру из второго столбца и запишите в таблицу выбранные цифры под соответствующей буквой.

А. Перемещение 1).

Б. Работа 2). м

В. Давление 3). Дж

4). Па

5). Вт

12) . Установите соответствие между физическими величинами и формулами, по которым эти величины определяются.

К каждой позиции первого столбца выберите соответствующую цифру из второго столбца и запишите в таблицу выбранные цифры под соответствующей буквой.

А. Сила тяжести 1)

Б. Плотность вещества 2) mgh

В. Потенциальная энергия 3) mυ

4) mg

При выполнении задания 10 решение запишите на отдельном бланке. Сначала укажите номер задания, а затем запишите его решение и ответ. Пишите чётко и разборчиво

  1. Сколько потребуется железнодорожных цистерн, чтобы перевезти 1000т нефти, если вместимость каждой цистерны 50 м3?

Ответы к заданиям варианта 1

задания

Ответы

1

В

2

А

3

В

4

В

5

Б

6

А

7

В

8

Б

9

Б

10

Б

11

412

12

254

13

426 км

Ответы к заданиям варианта 2

задания

Ответы

1

Б

2

А

3

Б

4

Б

5

В

6

А

7

А

8

В

9

А

10

А

11

234

12

412

13

23

Контрольная работа по физике Молекулярная физика 10 класс

Контрольная работа по физике Молекулярная физика 10 класс с ответами. Контрольная работа представлена в 5 вариантах, в каждом варианте по 8 заданий.

Вариант 1

A1. «Расстояние между соседними частицами вещества мало (они практически соприкасаются)». Это утверждение соответствует модели

1) только твердых тел
2) только жидкостей
3) твердых тел и жидкостей
4) газов, жидкостей и твердых тел

А2. При неизменной концентрации частиц идеального газа средняя кинетическая энергия теплового движения его молекул увеличилась в 3 раза. При этом давление газа

1) уменьшилось в 3 раза
2) увеличилось в 3 раза
3) увеличилось в 9 раз
4) не изменилось

А3. Чему равна средняя кинетическая энергия хаотического поступательного движения молекул идеального газа при температуре 27 °С?

1) 6,2 · 10-21 Дж
2) 4,1 · 10-21 Дж
3) 2,8 · 10-21 Дж
4) 0,6 · 10-21 Дж

А4. Какой из графиков, изображенных на рисунке, соответствует процессу, проведенному при постоянной температуре газа?

1) А
2) Б
3) В
4) Г

А5. При одной и той же температуре насыщенный пар в закрытом сосуде отличается от ненасыщенного пара в таком же сосуде

1) давлением
2) скоростью движения молекул
3) средней энергией хаотического движения
4) отсутствием примеси посторонних газов

B1. На рисунке показан график изменения давления идеального газа при его расширении.

Какое количество газообразного вещества (в молях) содержится в этом сосуде, если температура газа равна 300 К? Ответ округлите до целого числа.

В2. В сосуде неизменного объема находилась при комнатной температуре смесь двух идеальных газов, по 2 моль каждого. Половину содержимого сосуда выпустили, а затем добавили в сосуд 2 моль первого газа. Как изменились в результате парциальные давления газов и их суммарное давление, если температура газов в сосуде поддерживалась постоянной? К каждой позиции первого столбца подберите нужную позицию второго.

ФИЗИЧЕСКИЕ ВЕЛИЧИНЫ

А) парциальное давление первого газа
Б) парциальное давление второго газа
В) давление газа в сосуде

ИХ ИЗМЕНЕНИЕ

1) увеличилось
2) уменьшилось
3) не изменилось

C1. Поршень площадью 10 см2 может без трения перемещаться в вертикальном цилиндрическом сосуде, обеспечивая при этом его герметичность. Сосуд с поршнем, заполненный газом, покоится на полу неподвижного лифта при атмосферном давлении 100 кПа, при этом расстояние от нижнего края поршня до дна сосуда 20 см. Когда лифт поедет вверх с ускорением равным 4 м/с2, поршень сместится на 2,5 см. Какова масса поршня, если изменение температуры можно не учитывать?

Вариант 2

A1. «Расстояние между соседними частицами вещества в среднем во много раз превышает размеры самих частиц». Это утверждение соответствует

1) только модели строения газов
2) только модели строения жидкостей
3) модели строения газов и жидкостей
4) модели строения газов, жидкостей и твердых тел

А2. При неизменной концентрации молекул идеального газа средняя кинетическая энергия теплового движения его молекул изменилась в 4 раза. Как изменилось при этом давление газа?

1) в 16 раз
2) в 2 раза
3) в 4 раза
4) не изменилось

А3. При какой температуре средняя кинетическая энергия поступательного движения молекул газа равна 6,21 · 10-21 Дж?

1) 27 °С
2) 45 °С
3) 300 °С
4) 573 °С

А4. На рисунке показаны графики четырех процессов изменения состояния идеального газа. Изохорным нагреванием является процесс

1) А
2) В
3) С
4) D

А5. При одной и той же температуре насыщенный водяной пар в закрытом сосуде отличается от ненасыщенного пара

1) концентрацией молекул
2) средней скоростью хаотического движения молекул
3) средней энергией хаотического движения
4) отсутствием примеси посторонних газов

B1. Два сосуда, наполненные воздухом под давлением 800 кПа и 600 кПа, имеют объемы 3 ли 5 л соответственно. Сосуды соединяют трубкой, объемом которой можно пренебречь по сравнению с объемами сосудов. Найдите установившееся в сосудах давление. Температура постоянна.

В2. Установите соответствие между названием физической величины и формулой, по которой ее можно определить.

НАЗВАНИЕ

А) количество вещества
Б) масса молекулы
В) число молекул

ФОРМУЛА

1) m/V
2) ν · NА
3) m/NА
4) m/M
5) N/V

C1. Поршень площадью 10 см2 массой 5 кг может без трения перемещаться в вертикальном цилиндрическом сосуде, обеспечивая при этом его герметичность. Сосуд с поршнем, заполненный газом, покоится на полу неподвижного лифта при атмосферном давлении 100 кПа, при этом расстояние от нижнего края поршня до дна сосуда 20 см. Каким станет это расстояние, когда лифт поедет вниз с ускорением равным 3 м/с2? Изменение температуры газа не учитывать.

Вариант 3

A1. «Частицы вещества участвуют в непрерывном тепловом хаотическом движении». Это положение молекулярно-кинетической теории строения вещества относится к

1) газам
2) жидкостям
3) газам и жидкостям
4) газам, жидкостям и твердым телам

А2. Как изменится давление идеального одноатомного газа при увеличении средней кинетической энергии теплового движения его молекул в 2 раза и уменьшении концентрации молекул в 2 раза?

1) увеличится в 4 раза
2) уменьшится в 2 раза
3) уменьшится в 4 раза
4) не изменится

А3. Чему равна средняя кинетическая энергия хаотического поступательного движения молекул идеального газа при температуре 327 °С?

1) 1,2 · 10-20 Дж
2) 6,8 · 10-21 Дж
3) 4,1 · 10-21 Дж
4) 7,5 кДж

А4. На -диаграмме приведены графики изменения состояния идеального газа. Изобарному процессу соответствует линия графика

1) А
2) Б
3) В
4) Г

А5. В сосуде, содержащем только пар и воду, поршень двигают так, что давление остается постоянным. Температура при этом

1) не изменяется
2) увеличивается
3) уменьшается
4) может как уменьшаться, так и увеличиваться

B1. Два сосуда с объемами 40 ли 20 л содержат газ при одинаковых температурах, но разных давлениях. После соединения сосудов в них установилось давление 1 МПа. Каково было начальное давление в большем сосуде, если начальное давление в меньшем сосуде 600 кПа? Температуру считать постоянной.

В2. В сосуде неизменного объема находилась при комнатной температуре смесь двух идеальных газов, по 2 моль каждого. Половину содержимого сосуда выпустили, а затем добавили в сосуд 2 моль второго газа. Как изменились в результате парциальные давления газов и их суммарное давление, если температура газов в сосуде поддерживалась постоянной? К каждой позиции первого столбца подберите нужную позицию второго.

ФИЗИЧЕСКИЕ ВЕЛИЧИНЫ

А) парциальное давление первого газа
Б) парциальное давление второго газа
В) давление газа в сосуде

ИХ ИЗМЕНЕНИЕ

1) увеличилось
2) уменьшилось
3) не изменилось

C1. Поршень массой 5 кг может без трения перемещаться в вертикальном цилиндрическом сосуде, обеспечивая при этом его герметичность. Сосуд с поршнем, заполненный газом, покоится на полу неподвижного лифта при атмосферном давлении 100 кПа, при этом расстояние от нижнего края поршня до дна сосуда 20 см. Когда лифт поедет вниз с ускорением равным 2 м/с2, поршень сместится на 1,5 см. Какова площадь поршня, если изменение температуры газа не учитывать?

Вариант 4

А1. В жидкостях частицы совершают колебания возле положения равновесия, сталкиваясь с соседними частицами. Время от времени частица совершает прыжок к другому положению равновесия. Какое свойство жидкостей можно объяснить таким характером движения частиц?

1) малую сжимаемость
2) текучесть
3) давление на дно сосуда
4) изменение объема при нагревании

А2. В результате охлаждения одноатомного идеального газа его давление уменьшилось в 4 раза, а концентрация молекул газа не изменилась. При этом средняя кинетическая энергия теплового движения молекул газа

1) уменьшилась в 16 раза
2) уменьшилась в 2 раза
3) уменьшилась в 4 раза
4) не изменилась

А3. Средняя кинетическая энергия поступательного движения молекул газа в баллоне равна 4,14 · 10-21 Дж. Чему равна температура газа в этом баллоне?

1) 200 °С
2) 200 К
3) 300 °С
4) 300 К

А4. На рисунке показан цикл, осуществляемый с идеальным газом. Изобарному нагреванию соответствует участок

1) АВ
2) DA
3) CD
4) BC

А5. При уменьшении объема насыщенного пара при постоянной температуре его давление

1) увеличивается
2) уменьшается
3) для одних паров увеличивается, а для других уменьшается
4) не изменяется

B1. На рисунке показан график зависимости давления газа в запаянном сосуде от его температуры.

Объем сосуда равен 0,4 м3. Сколько молей газа содержится в этом сосуде? Ответ округлите до целого числа.

В2. Установите соответствие между названием физической величины и формулой, по которой ее можно определить.

НАЗВАНИЕ

А) концентрация молекул
Б) число молекул
В) масса молекулы

ФОРМУЛА

1) m/V
2) ν · NА
3) m/NА
4) m/M
5) N/V

C1. Поршень площадью 15 см2 массой 6 кг может без трения перемещаться в вертикальном цилиндрическом сосуде, обеспечивая при этом его герметичность. Сосуд с поршнем, заполненный газом, покоится на полу неподвижного лифта при атмосферном давлении 100 кПа. При этом расстояние от нижнего края поршня до дна сосуда 20 см. Когда лифт начинает двигаться вверх с ускорением, поршень смещается на 2 см. С каким ускорением движется лифт, если изменение температуры газа можно не учитывать?

Вариант 5

A1. Наименьшая упорядоченность в расположении частиц характерна для

1) газов
2) жидкостей
3) кристаллических тел
4) аморфных тел

А2. Как изменится давление идеального одноатомного газа, если среднюю кинетическую энергию теплового движения молекул и концентрацию уменьшить в 2 раза?

1) Увеличится в 4 раза
2) Уменьшится в 2 раза
3) Уменьшится в 4 раза
4) Не изменится

А3. При какой температуре средняя кинетическая энергия поступательного движения молекул газа равна 6,21 · 10-21 Дж?

1) 27 К
2) 45 К
3) 300 К
4) 573 К

А4. На рисунке показан цикл, осуществляемый с идеальным газом. Изобарному охлаждению соответствует участок

1) АВ
2) DA
3) CD
4) BC

А5. В сосуде под поршнем находятся только насыщенные пары воды. Как будет меняться давление в сосуде, если начать сдавливать пары, поддерживая температуру сосуда постоянной?

1) Давление будет постоянно расти
2) Давление будет постоянно падать
3) Давление будет оставаться постоянным
4) Давление будет оставаться постоянным, а затем начнет падать

B1. На рисунке. показан график изотермического расширения водорода.

Масса водорода 40 г. Определите его температуру. Молярная масса водорода 0,002 кг/моль. Ответ округлите до целого числа.

В2. Установите соответствие между названием физической величины и формулой, по которой ее можно определить.

НАЗВАНИЕ

А) плотность вещества
Б) количество вещества
В) масса молекулы

ФОРМУЛА

1) N/V
2) ν · NА
3) m/NА
4) m/M
5) m/V

C1. Поршень площадью 10 см2 массой 5 кг может без трения перемещаться в вертикальном цилиндрическом сосуде, обеспечивая при этом его герметичность. Сосуд с поршнем, заполненный газом, покоится на полу неподвижного лифта при атмосферном давлении 100 кПа, при этом расстояние от нижнего края поршня до дна сосуда 20 см. Каким станет это расстояние, когда лифт поедет вверх с ускорением равным 2 м/с2? Изменение температуры газа не учитывать.

Ответы на контрольную работу по физике Молекулярная физика 10 класс
Вариант 1
А1-3
А2-2
А3-1
А4-3
А5-1
В1. 20 моль
В2. 123
С1. 5,56 кг
Вариант 2
А1-1
А2-3
А3-1
А4-3
А5-1
В1. 675 кПа
В2. 432
С1. 22,22 см
Вариант 3
А1-4
А2-4
А3-1
А4-1
А5-1
В1. 1,2 МПа
В2. 213
С1. 9,3 см2
Вариант 4
А1-2
А2-3
А3-2
А4-1
А5-4
В1. 16 моль
В2. 523
С1. 3,89 м/с2
Вариант 5
А1-1
А2-3
А3-3
А4-3
А5-3
В1. 301 К
В2. 543
С1. 18,75 см

Контрольная работа по физике (9 класс)

Контрольная работа по физике (9 класс)

Вариант 1

Часть А

1. Велосипедист движется из точки А велотрека в точку В по кривой АВ. Назовите
физическую величину, которую изображает вектор АВ.

А

В А) путь; Б) перемещение; В) скорость. Г) ускорение

2. Тело движется равномерно по окружности в направлении против часовой стрелки. Какая стрелка указывает направление вектора скорости тела в точке 1?

2

1

1 3 А) 1; Б) 2; В) 3; Г) 4.

4

3. Автомобиль, движущийся прямолинейно равноускоренно, увеличил свою скорость с
3 м/с до 9 м/с за 6 секунд. С каким ускорением двигался автомобиль?

А) 0 м/с2; Б) 1 м/с2; В) 2 м/с2; Г) 3 м/с2.

4. По графику зависимости скорости от времени определите ускорение тела в момент времени

t = 2 с.

v (м/с)

А) 1 м/с2; Б) 2 м/с2; В) 1,5 м/с2. Г) 0,5 м/с2

6

4

2

0 1 2 3 t (с)

5. Уравнение зависимости проекции скорости движения тела от времени vх = 2 + 3t. Каким будет соответствующее уравнение проекции перемещения?

A) Sx= 2t + l,5t2; Б) Sx = 2t + 3t2; B) Sx= l,5t2; Г) Sх = 3t + t2.

6. Как изменится сила трения скольжения при движении бруска по горизонтальной
поверхности, если силу нормального давления увеличить в 2 раза?

А) не изменится; Б) увеличится в 2 раза;

В) уменьшится в 2 раза; Г) увеличится в 4 раза.

7. При столкновении двух вагонов буферные пружины жесткостью 105 Н/м сжались
на 10 см. Чему равна максимальная сила упругости?

А) 104 Н; Б) 2 ∙ 104 Н; В) 106 Н; Г) 2 ∙ 106 Н.

8. Какая из приведенных формул выражает закон всемирного тяготения?

A) ; Б) ; B) ; Г) .

9. Конькобежец массой 60 кг скользит по льду. Определите силу трения скольжения,
действующую на конькобежца, если коэффициент трения скольжения равен 0,015.
А) 400 Н; Б) 40 Н; В) 9 Н; Г) 0,9 Н.

10. На рисунке показаны направления скорости и ускорения тела в данный момент времени. Какая из стрелок (1-4) на рис.Б соответствует направлению результирующей всех сил, действующих на тело.

А) 1 Б) 2 В) 3 Г) 4

Часть В

1. На дне шахтной клети лежит груз массой 100 кг. Каким будет вес груза, если клеть
поднимается вверх с ускорением 0,3 м/с2? (g ≈ 9,8 м/с2)

2. Рудничный электровоз движет вагонетки с ускорением 0,1 м/с2. Масса электровоза с вагонетками 90 т. Сила сопротивления движению 4000 Н. Найдите силу тяги.

Критерии оценивания.

А1 – А10 по 1 баллу

В1 – В2 по 2 балла.

«5» — 12 – 14 баллов

«4» — 9 – 11 баллов

«3» — 6 – 8 баллов

«2» — 0 – 5 баллов

Ответы:

Вариант1

Часть А Часть В

1. Б 6. Б 1. 1010 Н

2. Г 7. А 2. 13000 Н

3. Б 8. В

4. Б 9. В

5. А 10. Б

Контрольная работа по физике (9 класс)

Вариант 2

Часть А

1. Какие из перечисленных величин являются скалярными?

А) скорость; Б) путь; В) перемещение. Г) ускорение

2. Тело движется равномерно по окружности. Какая стрелка указывает направление вектора ускорения тела в точке М траектории?

2

М

1 3 А) 1; Б) 2; В) 3; Г) 4.

4

3. По графику зависимости пройденного пути от времени при равномерном движении определите скорость велосипедиста в момент времени t = 2 с.

S (м)

6

4 А) 2 м/с; Б) 3 м/с; В) 6 м/с; Г) 18 м/с.

2

0 1 2 3 t (с)

4. При отходе от станции ускорение поезда составляет 1 м/с2. Какой путь проходит поезд за 10 с?

А) 5 м; Б) 10 м; В) 50 м; Г) 100 м.

5. По уравнению координаты движения автомобиля х = 100 + 4t – 3t2 определите ускорение ах его движения.

А) 4 м/с2; Б) 3 м/с2; В) -6 м/с2; Г) -3 м/с2.

6. По горизонтальной плоскости равномерно движется брусок массой m. Чему равен
модуль силы трения, действующей на брусок?

A) µmg; Б) mg; B) 0; Г) µmgcos α.

7. На рисунке представлены графики зависимости модулей сил упругости от деформации для трех пружин. Жесткость какой пружины больше?

Fх 1 А) 3;

2 Б) 2;

В) 1.

3 Г) все равны

0 х

8. Какая сила сообщает ускорение свободного падения стреле, выпущенной из лука?

А) сила сопротивления; Б) сила упругости; В) сила тяжести; Г) вес тела.

9. Тело массой 10 кг движется равномерно прямолинейно по горизонтальной поверхности под действием силы 20 Н, направленной горизонтально. Каково значение коэффициента трения?

А) 20; Б) 2; В) 0,5; Г) 0,2.

10. При увеличении в 3 раза расстояния между центрами шарообразных тел сила гравитационного притяжения

А) увеличивается в 3 раза

Б) уменьшается в 3 раза

В) увеличивается 9 раз

Г) уменьшается в 9 раз

Часть В

1. На дне шахтной клети лежит груз массой 100 кг. Каким будет вес груза, если клеть

опускается вниз с ускорением 0,4 м/с2? (g =10 м/c2)

2. Брус массой 5 кг, лежащий на горизонтальной поверхности, начал двигаться с ускорением 1 м/с2 под действием горизонтальной силы F = 30 Н. Чему равен коэффициент трения?

Критерии оценивания.

А1 – А10 по 1 баллу

В1 – В2 по 2 балла.

«5» — 12 – 14 баллов

«4» — 9 – 11 баллов

«3» — 6 – 8 баллов

«2» — 0 – 5 баллов

Ответы:

Вариант 2

Часть А Часть В

1. Б 6. А 1. 960 Н

2. В 7. В 2. 0,5

3. А 8. В

4. В 9. Г

5. В 10. Г

Контрольная работа по физике (9 класс)

Вариант 3

Часть А

1. Какая из приведенных формул соответствует определению скорости?

A) ; Б) ; В) ; Г) .

2. У верхнего конца трубки, из которой выкачан воздух, находятся дробинка, пробка, птичье перо. Какое из этих тел при одновременном старте первым достигает нижнего конца трубки?

А) дробинка; Б) пробка; В) перо; Г) все тела.

3. На графике изображена зависимость координаты х от времени. Чему равна начальная координата тела?

х (м)

А) 0; Б) 1 м; В) -1 м; Г) -2 м.

2

1

0

-1 1 2 3 t (с)

4. Уравнение координаты движения автомобиля имеет вид: х = 100 + 4t — 3t2. Чему равна координата автомобиля в начальный момент времени?

А) 4 м; Б) 3 м; В) 100 м; Г) -3 м.

5. По графику зависимости модуля скорости от времени, представленному на рисунке, определите перемещение тела за 3 секунды.

v(м/с)

А) 9 м; Б) 18 м; В) 27 м; Г) 36 м.

9

6

3

0 1 2 3 t(с)

6. Какое равенство правильно выражает закон Гука для упругой пружины?

A) F = kx; Б) Fx = kx; В) Fх = — kx; Г) Fх = .

7. На рисунке приведен график зависимости модуля силы упругости от удлинения пружины. Чему равна жесткость пружины?

F,Н А) 10 Н/м; Б) 0,01 Н/м;

20 В) 0,02 Н/м; Г) 100 Н/м.

10

0 0,1 0,2 0,3 0,4 х,м

8. Что притягивает к себе с большей силой: Земля Луну или Луна Землю?

А) Земля Луну; Б) Луна Землю; В) одинаковые силы. Г) не притягиваются

9. Сила, прижимающая деревянный ящик к полу, 400 Н. Чтобы его сдвинуть с места,
потребовалось приложить силу 200 Н. Определите коэффициент трения покоя.

А) 1,5; Б)1; В) 0,5; Г) 2.

10. Под действием равнодействующей силы, равной 5 Н, тело массой 10 кг движется

А) равномерно со скоростью 2 м/с

Б) равномерно со скоростью 0,5 м/с

В) равноускоренно с ускорением 2 м/с2

Г) равноускоренно с ускорением 0,5 м/с2

Часть В

1. На дне шахтной клети лежит груз массой 100 кг. Каким будет вес груза, если клеть свободно падает? (g ≈ 9,8 м/c2)

2. Определите тормозной путь автомобиля, начавшего торможение на горизонтальном
участке шоссе с коэффициентом трения 0,5 при начальной скорости 10 м/с.

Критерии оценивания.

А1 – А10 по 1 баллу

В1 – В2 по 2 балла.

«5» — 12 – 14 баллов

«4» — 9 – 11 баллов

«3» — 6 – 8 баллов

«2» — 0 – 5 баллов

Ответы:

Вариант 3

Часть А Часть В

1. В 6. В 1. О Н

2. Г 7. Г 2. 10 м

3. В 8. В

4. В 9. В

5. Б 10. Г

Контрольная работа по физике (9 класс)

Вариант 4

Часть А

1. . Какая из приведенных формул соответствует определению ускорения?

А) ; Б) ; В) ; Г) .

2. По графику зависимости скорости тела от времени определите пройденный телом путь за 2 с.

v (м/с) А) 20 м;

Б) 30 м;

В) 40 м;

20 Г) 10 м.

10

0 1 2 3 t (с)

3. По графику зависимости скорости тела от времени определите вид движения.

v (м/с)

А) равноускоренное; Б) равнозамедленное;

В) прямолинейное; Г) равномерное.

40

20

0 1 2 3 t (с

4. При равноускоренном движении в течение 5 с автомобиль увеличил скорость от 10 до

15 м/с. Чему равен модуль ускорения автомобиля?

А) 1 м/с2; Б) 2 м/с2; В) 3 м/с2; Г) 5 м/с2.

5. Троллейбус, трогаясь с места, движется с постоянным ускорением 1,5 м/с2. Через какое время он приобретет скорость 54 км/ч?

А) 5 с; Б) 6 с; В) 10 с; Г) 2 с.

6. По какой из перечисленных формул можно определить силу трения?

A) F = mg; Б) F = µN; B) F = mgcosα. Г) F = — kx

7. Брусок массой 0,2 кг равномерно тянут с помощью горизонтально расположенного
динамометра по горизонтальной поверхности стола. Показания динамометра 0,5 Н. Чему
равен коэффициент трения?

А) 0,2; Б) 0,25; В) 0,4; Г) 0,5.

8. Тело, изображенное на рисунке, находится в покое. Определите условие равновесия
этого тела.

А) Fтяж > Fупр; Б) Fтяжупр;

В) Fтяж = Fупр; Г) Fупр = 0.

9. На рисунке приведена зависимость модуля силы трения от силы нормального давления N. Определите по графику коэффициент трения.

Fтр

3 А) 0,25; Б) 0,5;

2 В) 0,75; Г) 0,1.

1

0 1 2 3 4 N

10. Скорость лыжника при равноускоренном спуске с горы за 4 с увеличилась на 6 м/с. Масса лыжника 60 кг. Равнодействующая всех сил, действующих на лыжника, равна

А) 20 Н Б) 30 Н В) 60 Н Г) 90 Н

Часть В

1. Лифт поднимается с ускорением 2 м/с2, вектор ускорения направлен вертикально вверх. В лифте находится тело массой 1 кг. Чему равен вес тела (g = 10м/с2)?

2. После толчка тепловоза железнодорожный вагон останавливается, пройдя путь 37,5 м за 50 с. Каков коэффициент трения?

Критерии оценивания.

А1 – А10 по 1 баллу

В1 – В2 по 2 балла.

«5» — 12 – 14 баллов

«4» — 9 – 11 баллов

«3» — 6 – 8 баллов

«2» — 0 – 5 баллов

Ответы:

Вариант 4

Часть А Часть В

1. Б 6. Б 1. 12 Н

2. В 7. Б 2. 0,003

3. Г 8. В

4. А 9. Б

5. В 10. Г

Диагностическая итоговая контрольная работа по физике 7 класс

Диагностическая работа по физике 7класс 

Справочные данные, которые могут понадобиться вам при выполнении работы.

Десятичные приставки

Наименование Обозначение Множитель Наименование Обозначение Множитель

мега М 10 6 санти с 10– 2

кило к 10 3 милли м 10– 3

гекто г 10 2 микро мк 10 – 6

 

Константы

ускорение свободного падения на Земле g = 10 Н/кг

 

Плотность

бензин 710 кг/м3 парафин 900  кг/м3

нефть 800 кг/м3 древесина (сосна) 400 кг/м3

спирт 800 кг/м3 древесина (дуб) 800 кг/м3

масло машинное 900 кг/м3 лёд 900 кг/м3

вода 1000 кг/м3 алюминий, мрамор 2700 кг/м3

вода морская 1030 кг/м3 железо, сталь 7800 кг/м3

глицерин 1260 кг/м3 медь 8900 кг/м3

 

Нормальные условия     давление   105 Па,  температура  0С

 

Вариант 1

Часть 1

При выполнении заданий этой части (1-13),  из четырёх предложенных вариантов выберите один верный и перенесите в бланк ответов

1. Физическим телом является …:

1) самолет 2) вода 3) метр 4) кипение

2. Веществом является…:

1) килограмм 2) звук 3) алюминий 4) Земля

3. К звуковым явлениям относятся: 

1) шар катится;  2) слышны раскаты грома; 3) снег тает; 4) наступает рассвет

4. Молекулы льда и воды отличаются друг от друга:

1)  Формой 2) Размером  3) Молекулы одного и того же вещества в жидком и в твердом состояниях одинаковы

5. Явление диффузии доказывает…

1) Только факт существования молекул   

2) Только факт движения молекул.

3) Факт существования и движения молекул

6. Между молекулами любого вещества действуют

1) Только силы отталкивания 2) Только силы притяжения

3) Силы притяжения и отталкивания 4) Не действуют никакие силы

7. Жидкость:

1) Занимает объем всего сосуда 2) Легко поддается сжатию

3) Принимает форму сосуда

4) Имеют кристаллическое строение.

8.  Выразите массу, равную 0,05 кг, в граммах

1). 5 г        2). 50 г       3). 500 г      4). 5000 г

    9. Грузовик ехал 5 мин со  средней скоростью 36 км/ч. Какой путь он проехал за это время?

1). 600 м     2). 1000 м     3). 3000 м     4). 1500 м

10.  Найдите объём  парафина  массой 0,27 кг .

 1). 0,0003 м3                2). 27 м3           3). 33 м3              4). 3,3 м3      

   11.  Определите плотность асфальта, если 4м3 этого вещества имеют массу 8000 кг.

1). 1200 кг/м3             2). 1500 кг/м3             3). 1800 кг/м3                4). 2000 кг/м3                   

На какой глубине давление воды равно 400 кПа?

1) 200 м;

2) 40 м;

3) 400 м;

4) 1000 м;

К чашам весов подвешены два шарика равной массы из различных веществ (ρ1> ρ2). Нарушится ли равновесие весов, если шарики одновременно опустить в масло?

  

1) перетянет шарик 1;

2) перетянет шарик 2;

3) равновесие не нарушится;

4) среди предложенных ответов нет верного;

 

Ответом к заданию 14, 15 является последовательность цифр. При записи  ответа в бланке ответов следует указать только эту последовательность, без запятых, пробелов и прочих символов

 

14.  Установите соответствие между физическими величинами и основными единицами их измерения в СИ. К каждой позиции первого столбца подберите соответствующую позицию второго и запишите в таблицу выбранные цифры под соответствующими буквами.

ФИЗИЧЕСКАЯ ВЕЛИЧИНА ЕДИНИЦА ИЗМЕРЕНИЯ

А) масса 1) грамм (г)

Б) скорость 2) м/с

3) килограмм (кг)

4) км/ч

             

            А Б

15.Для каждого физического понятия из первого столбца подберите соответствующий пример из второго столбца.

 

ФИЗИЧЕСКИЕ ПОНЯТИЯ

ПРИМЕРЫ

 

А) физическая величина

Б) единица физической величины

В) физическое явление 1) атмосферное давление

2) мензурка

3) диффузия

4) Паскаль

5) манометр

 

            А Б

 

Часть 2

Решите задачу и укажите ответ цифрами в указанных единицах измерения

 

Определите глубину погружения батискафа, если на его иллюминатор площадью 0,12 м2 давит вода с силой 1,2 МН. (Ответ дайте в м).

 

Чему равна сила архимеда, действующая на  мраморную балку объемом 6 м3 в воде?  

Диагностическая работа по физике 7класс за 2018-2019 учебный год

 

Справочные данные, которые могут понадобиться вам при выполнении работы.

Десятичные приставки

Наименование Обозначение Множитель Наименование Обозначение Множитель

мега М 10 6 санти с 10– 2

кило к 10 3 милли м 10– 3

гекто г 10 2 микро мк 10 – 6

 

Константы

ускорение свободного падения на Земле g = 10 Н/кг

 

Плотность

бензин 710 кг/м3 парафин 900  кг/м3

нефть 800 кг/м3 древесина (сосна) 400 кг/м3

спирт 800 кг/м3 древесина (дуб) 800 кг/м3

масло машинное 900 кг/м3 лёд 900 кг/м3

вода 1000 кг/м3 алюминий, мрамор 2700 кг/м3

вода морская 1030 кг/м3 железо, сталь 7800 кг/м3

глицерин 1260 кг/м3 медь 8900 кг/м3

 

Нормальные условия     давление   105 Па,  температура  0С

 

Вариант 2

Часть 1

При выполнении заданий этой части (1-13),  из четырёх предложенных вариантов выберите один верный и перенесите в бланк ответов

1. Веществом является…. 

1) автомобиль 2) вода 3) звук 4) кипение

2. Физической величиной является.

1) самолет 2) алюминий 3) время 4) мензурка

3. Физическим телом является …

1) скорость 2) кипение 3) метр 4) авторучка

4. Все вещества состоят

1) только из нейтронов 2) только из протонов

3) молекул, атомов и других частиц 4) только из электронов

5. Диффузия протекает быстрее

1) в твердых телах; 2) в жидких телах;  3) в газах; 4) одинаково во всех

 

6.Твердое тело:

1) Занимает объем всего сосуда 2) Легко поддается сжатию; 3)Имеет кристаллическое строение

   7.  Выразите  скорость   54  км/ч  в  м/с.   

1). 20 м/с       2). 15 м/с        3). 130,6 м/с       4). 0,54 м/с 

  8. Через сколько  времени акула  приплывет к берегу, если она находится от него на расстоянии 8 км и её скорость равна 40 км/ч?

1). 12 мин       2). 0,5 ч        3). 24 мин        4). 0,2 ч      

9. Какова  плотность  горной  породы, осколок  которой  объёмом 0,03 м3  обладает  массой  81 кг?

1). 2700 кг/м3        2). 270 кг/м3         3). 243 кг/м3           4)  2430 кг/м3                     

       

10.      Для  изготовления  ледяной скульптуры был использован кусок льда массой 324 кг. Определите объём  куска льда. 

1). 3,5 м3                            2). 16 м3            3). 36 м3                                 4). 0.36 м3

В каких единицах измеряется давление?

1) Джоуль;

2) Ньютон;

3) Ватт;

4) Паскаль;

Высота столба воды в стакане 8 см. Какое давление на дно стакана оказывает вода?

1) 1000 Па;

2) 800 Па;

3) 600 Па;

4) 20 Па;

Четыре одинаковых стакана заполнили разными жидкостями. (См. рисунок). Высота уровней жидкостей одинакова. В каком из сосудов давление на дно наибольшее?

 1) в стакане 1;

                                          2) в стакане 2;

                                       3) в стакане 3;

                                       4) в стакане 4;

 

Ответом к заданию 14, 15 является последовательность цифр. При записи  ответа в бланке ответов следует указать только эту последовательность, без запятых, пробелов и прочих символов


 

14.   Установите соответствие между физическими величинами и приборами, с помощью которых  их  можно  измерить. К каждой позиции первого столбца подберите соответствующую позицию второго и запишите в таблицу выбранные цифры под соответствующими буквами.

 

ПРИБОР ФИЗИЧЕСКАЯ ВЕЛИЧИНА

А) мензурка 1) вес

Б) весы 2) масса

3) плотность

4) объём

А Б

Установите соответствие между физическими величинами и единицами измерения этих величин в СИ:

 

ФИЗИЧЕСКИЕ ВЕЛИЧИНЫ

ЕДИНИЦЫ ИЗМЕРЕНИЯ

 

А) давление

Б) сила

В) плотность

1) кг/м^3

2) Н

3)м^3/кг

4) Па

5) Дж

            А Б

 

Часть 2

Решите задачу и укажите ответ цифрами в указанных единицах измерения

 

16. Какова масса трактора, если опорная площадь его гусениц равна 1,3 м2, а давление на почву составляет 40 кПа? (Ответ дайте в кг).

 

17.  Чему равна архимедова сила, действующая на тело объемом 2 м3, находящемся в воде? 

Вес каждого задания при подсчете результата: часть А – 1 балл, часть В – 2 балла, 1 балл- 1 не правильно, часть С – 3 балла. Максимальное количество баллов за работу -26.

 

Рекомендуемые отметки:

 

7 и ниже баллов — отметка «2»;

 

от 8 до 12 баллов — отметка «3»;

 

от 13 до 17 баллов — отметка «4»;

 

от 18 до 23 баллов — отметка «5».

Ответы к диагностической работе :

 

№ задания

1

2

3

4

5

6

7

8

9

10

11

12 13 14 15 16 17

вариант 1 1 3 2 3 3 3 3 2 3 1 4 2 1 32 143 1000 60 000Н

Вариант 2 2 3 4 4 3 3 2 1 1 4 4 2 2 42 421 5200 кг 20 000 Н

▶▷▶▷ ответы на контрольную работу по физике 8 класс вариант 3

▶▷▶▷ ответы на контрольную работу по физике 8 класс вариант 3
ИнтерфейсРусский/Английский
Тип лицензияFree
Кол-во просмотров257
Кол-во загрузок132 раз
Обновление:12-08-2019

ответы на контрольную работу по физике 8 класс вариант 3 — Контрольная работа по физике Электрические явления 8 класс testschoolru20171021kontrolnaya-rabota-po Cached Контрольная работа по физике Электрические явления для учащихся 8 класса с ответами Тест включает в себя 4 варианта, в каждом по 8 заданий Контрольная работа по физике Оптика 11 класс testschoolru20171206kontrolnaya-rabota-po Cached Контрольная работа по физике Оптика для учащихся 11 класса с ответами Контрольная работа включает 5 вариантов, в каждом варианте по 8 заданий Ответы На Контрольную Работу По Физике 8 Класс Вариант 3 — Image Results More Ответы На Контрольную Работу По Физике 8 Класс Вариант 3 images Физика 7 класс Контрольные работы (Перышкин) контрольз на нийрфfizika-7-klass Cached Тест по по теме Физика 7 класс Контрольные работы (Перышкин) Проверка ответа стразу Годовая контрольная работа по физике 8 класс multiurokrufilesgodovaia-kontrol-naia-rabota Cached Итоговая контрольная работа по физике для 8 класса 2 вариант А1 Каким способом можно изменить внутреннюю энергию тела? 1) только совершением работы 2) только теплопередачей 3 ) совершением Физика 8 Итоговая контрольная работа ОТВЕТЫ, ГДЗ всеконтрольныерфfizika-8-itogovaja Cached Итоговая контрольная по физике 8 класс (УМК любой, автор ИВ Годова) ОТВЕТЫ Физика 8 Итоговая контрольная это годовая контрольная работа и ОТВЕТЫ к ней (цитаты) из пособия Физика 8 класс Контрольные работы по физике 7 класс Итоговая контрольная работа onlyegerukontrolnaya-rabota-po-fizike-7-klass Cached Контрольные работы по физике 7 класс Итоговая контрольная работа по физике 7 класс Вариант приготовила Султанова Н Контрольная работа по физике Световые явления 7 класс testytutru20190407kontrolnaya-rabota-po Cached Ответы на контрольную работу по физике Световые явления 7 класс Вариант 3 2 1, 8 м Контрольные работы по физике 8 класс samopodgotovkacomindexphpfizika25-kontrolnye-raboty Cached Вариант 1 Контрольная работа по физике 8 класс Тема: Постоянный ток Вариант 2 Контрольная работа по физике 8 класс Тема: Постоянный ток Вариант 3 Контрольная работа по физике 8 класс Контрольные и самостоятельные работы по физике 8 класс к allengorgdphysphys416htm Cached Оно ориентировано на учебник АВ Перышкина Физика 8 класс и содержит контрольные работы по всем темам, изучаемым в 8 классе, а также самостоятельные работы к каждому параграфу Итоговая контрольная работа по физике 7 класс 2 варианта с infourokruitogovaya-kontrolnaya-rabota-po Cached Итоговая контрольная работа по физике 7 класс 2 варианта с ответами от проекта Инфоурок Отдых и оздоровление в экологически чистом Promotional Results For You Free Download Mozilla Firefox Web Browser wwwmozillaorg Download Firefox — the faster, smarter, easier way to browse the web and all of 1 2 3 4 5 Next 25,000

  • ответы на
  • контрольную работу п
  • ю работу по физике 8 класс вариант 3

  • easier way to browse the web and all of 1 2 3 4 5 Next 25
  • ГДЗ всеконтрольныерфfizika-8-itogovaja Cached Итоговая контрольная по физике 8 класс (УМК любой
  • в каждом по 8 заданий Контрольная работа по физике Оптика 11 класс testschoolru20171206kontrolnaya-rabota-po Cached Контрольная работа по физике Оптика для учащихся 11 класса с ответами Контрольная работа включает 5 вариантов

Нажмите здесь , если переадресация не будет выполнена в течение нескольких секунд ответы на контрольную работу по физике класс вариант Поиск в Все Картинки Ещё Видео Новости Покупки Карты Книги Все продукты ФИЗИКА КЛАСС КОНТРОЛЬНЫЕ РАБОТЫ В НОВОМ zvonoknaurokru fizika _ _ klass ВАРИАНТ В разделе размещены контрольные работы по всем темам традиционного курса физики класса Контрольная по физике класс Инфоурок май Контрольная работа по физике класс Вариант воды Джкг С; удельная теплота парообразования воды , Джкг Ответы к контрольной работе Контрольная работа по физике класс Инфоурок ноя Рубежная контрольная работа по физике в классе К каждому заданию дано несколько ответов , из которых только один верный Вариант а А; б А; в А; г , ГДЗ по физике класс контрольные и самостоятельные eurokiorg fizika _ klass ГДЗ контрольные и самостоятельные работы по физике класс Громцева Экзамен Многие восьмиклассники ГДЗ по физике класс самостоятельные и контрольные eurokiorg fizika _ klass ГДЗ самостоятельные и контрольные работы по физике класс Марон Дрофа Физика одна из сложных Итоговая контрольная работа по физике класс АВ fizika Скачать Итоговая контрольная работа по физике класс АВ Работа состоит из частей, включающих вариант вопроса А А А А А А А А Ответ Контрольные работы по физике класс Итоговая onlyegerukontrolnayarabotapofizike апр Итоговая контрольная работа по физике класс Вариант приготовила Султанова Н Вариант Вычислите силу тока в стальном проводнике ОТВЕТЫ В А А В В Физика Контрольные Перышкин КОНТРОЛЬНЫЕ РАБОТЫ все контрольные рф fizika kontrolnye Контрольные работы по физике класс УМК Перышкин АВ Физика КР ОТВЕТЫ на все варианта КР Постоянный ток В Физика Перышкин Контрольная работа контрольные работы по физике для класса Социальная fizika kontrolnye дек картотека по физике класс по теме Представлено по варианта контрольных работ по физике класса по Контрольная работа Готовые тесты по физике для класса с ответами , варианта Итоговая Контрольная Работа По Физике Класс Вариант мар Итоговая Контрольная Работа По Физике Класс Вариант В работу включено заданий, ответы к которым представлены в виде последовательности цифр, символов, Контрольные работы класс Мультиурок ноя класс к учебнику АВПерышкина Физика класс ОИГромцева е изд перераб и Тепловые явления В , Итоговая контрольная работа вариант класс контрольная работа по физике класс перышкин тепловые явления Лучший ответ Алёнка Ученик года назад physhtm Комментарий удален Физика класс Контрольные работы Перышкин контрользнанийрф fizika klass июл Физика класс Контрольные работы Перышкин ОТВЕТЫ Решения задач из учебного Тест по физике класс на тему Контрольная работа по fizika kontrolnaya янв Контрольная работа по теме Теплота и энергия баллов отметка Часть А открытых заданий содержит по варианта ответов , причём каждый вопрос Входная контрольная работа по физике класс авг с ответами Работа состоит из вариантов в каждом варианте части всего заданий Входная контрольная работа по физике класс с ответами Работа состоит из Итоговая контрольная работа по физике за класс , ФГОС апр Итоговая контрольная работа по физике за класс К каждому из заданий даны варианта ответа, из которых только один Ответ Контрольная работа по физике класс Тема Тепловые samopodgotovkacom fizika klass Контрольная работа по физике класс Тема Тепловые Изменение агрегатных состояний Вариант Ответы Контрольные и самостоятельные работы по физике по физике класс к учебнику Перышкина АВ Громцева ОИ pdf Контрольные и самостоятельные работы по физике класс к Вариант Вариант Глава Электромагнитные явления ОТВЕТЫ Контрольная работа по физике Тепловые явления класс Контрольная работа по физике Тепловые явления класс вариант Ответы на контрольную работу по физике Тепловые явления вариант С вариант г вариант , кг ГДЗ по физике за класс тематические контрольные class fizika tematicheskie ГДЗ Спиши готовые домашние задания тематические контрольные работы по физике за класс , решебник Контрольно измерительный материал по физике класс _ klass класс doc Диагностическая контрольная работа по физике класс диагностической контрольной работы Проверяемый материал Прочитайте вопросы и выберите варианты ответов , которые вы считаете Итоговая контрольная работа по физике класс fizika itogovaya Вариант А В жидкостях частицы совершают колебания возле положения равновесия, сталкиваясь с Физика класс Контрольная работа за первую fizika klass Материал из раздела контрольные работы от Физика класс Контрольная работа за первую PDF Контрольные и самостоятельные работы prutschoolucozru _ klass kontri_ работы по физике Кучебнику А В Перышкина Физика класс М Дрофа Вариант Вариант Вариант Вариант Вариант Вариант ОТВЕТЫ л е н и Решебник проверочные и контрольные работы по Физике class fizika Решебник ГДЗ проверочные и контрольные работы по Физике за класс ГДЗ к рабочей тетради по физике за класс Пурышева НС можно Проверочная работа Вариант Контрольная работа Количество теплоты класс дек Контрольная работа даётся в конце раздела Тепловые явления Вариант Дайте полный ответ Электронная тетрадь по физике класс класс KB Разное по физике класс Тесты по физике для го класса онлайн Online Test Pad class Итоговая контрольная работа по физике за курс класса Вариант Тест по теме Электрические явления I вариант Тест можно Подразумевает выбор одного ответы Тестовая олимпиада по физике для класса Итоговая контрольная работа по физике в классе за pedsovetsuload янв физика класс вариант к контрольной работе по физике в классе за первое полугодие Сказали спасибо человек, из них пользователя время появляется правильный ответ к очередному лепестку Картинки по запросу ответы на контрольную работу по физике класс вариант Итоговая контрольная работа по физике для учащихся wwwopen class runode май Итоговая контрольная работа по физике класс Учени_____ Вариант ЧАСТЬ А Ватт Вт В электрический заряд Ампер А Вольт В Ом Ом Ответ Контрольная работа по физике класс Тепловые явления ladlavnarodruf_k_r_htm Контрольная работа по теме Тепловые явления класс Вариант Удельная теплота сгорания пороха , Джкг Джкг С, удельная теплота сгорания бензина , Джкг Ответ примерно градусов Контрольные работы по физике классов физика , тесты fizika kontrol июн Контрольные работы по физике для классов кучебнику ПерышкинаКаждая работа состоит из вариантов , имеется Ответ на автомобиль можно погрузить кирпича ρ кг м Па , кПа ГДЗ Физика класс Громцева Контрольные и _ klass kontrolnyei В ГДЗ по физике класс Громцева можно найти подробнейшие ответы на каждый пункт проверочных тестов работа Вариант Вариант Вариант Вариант Контрольная работа Физика класс Контрольные и самостоятельные работы к labirintrureviews Интересные рецензии пользователей на книгу Физика класс Контрольные и самостоятельные работы к Контрольные работы ГДЗ по Физике для класса Марона klass дек Решебник по физике для учеников класса под Контрольные работы Расчет количества теплоты Вариант Q L m , Джкг , кг , Дж Варианты контрольной работы Электрические явления klass Контрольная работа по теме электрические явления Варианты ответов А или Б в любое В или Г или Рис Схема Фадеева АА, Засов АВ, Киселев ДФ Физика СРРРООООЧНО ДАМ баллов!!!! нужны ответы на дек СРРРООООЧНО ДАМ баллов!!!! нужны ответы на контрольную работу по физике , класс , Решебник по физике ЛА Кирик Самостоятельные и kupuknet klass reshebnikpofizikela ГДЗ решебник по физике классов Решебник по физике ЛА Кирик Самостоятельные и контрольные работы Самостоятельная работа Самостоятельная работа PDF Физика класс Демонстрационный вариант Итоговая schooladmsurgutru Демонстрационный вариант г Ответы к заданиям , записываются в виде одной цифры, которая для проведения итоговой контрольной работы по ФИЗИКЕ класс Физика класс Контрольная работа по теме Курсотека kursotekarucourse Электрический ток Физика Физика класс Контрольная работа по теме Учебный план; Отзывы ; Вопросы и ответы Вариант Длительность Контрольная работа Тест Контрольные работы по физике класс Контрольные reshuzadachibykontrolnyerabotypo ноя Контрольные работы по физике класс РАСЧЕТ КОЛИЧЕСТВА ТЕПЛОТЫ ПРИ НАГРЕВАНИИ И Контрольные и самостоятельные работы по физике , янв Физика класс и содержит контрольные работы по всем темам, Вариант ОТВЕТЫ Проверочная работа по физике klass proverochnaya Представлено по варианта контрольных работ по физике класса по учебнику Перышкин Контрольные работы Контрольная работа Бланк ответов заполняется с двух сторон Вводная контрольная работа по физике класс Документ работа по физике класс задания ответ задания ответ Контрольная работа по теме Световые явления Контрольная работа по теме Световые явления класс , На рисунке показан ход луча на вариант Ответы к тестам по физике класс Громцева SlideShare slidesharenet май Ответы к тестам по физике класс Громцева Электрические явления Контрольная работа Удельная теплота сгорания варианта МДж МДж МДжкг Контрольные работы Готовые домашние задания ГДЗ wwwmygdzcomdidakticheskie ГДЗ по алгебре, Дидактические материалы по алгебре, класс , Жохов В И Контрольные работы Готовое Контрольные и самостоятельные работы по физике работы по физике класс К учебнику Перышкина АВ Физика класс Контрольные и самостоятельные работы по физике класс К учебнику показать все Физика класс Кормаков НА Опорные конспекты Тесты class fizika narodrukormhtm дек Тесты Контрольные работы Класс!ная физика Материалы по физике для класса ОТВЕТЫ к тестам для самоконтроля смотреть Электрический ток Вариант , Вариант , Вариант , Вариант Контрольная работа тест по физике для промежуточной uchkopilkaru fizika kontrolnaya июл тест по физике для промежуточной аттестации за класс содержит Бланк ответов заполняется с двух сторон Ответы Решения задач части Варианты Запросы, похожие на ответы на контрольную работу по физике класс вариант контрольные и самостоятельные работы по физике класс ответы контрольные и самостоятельные работы по физике класс читать контрольные и самостоятельные работы по физике класс перышкин ответы контрольная работа по физике класс световые явления контрольная работа по физике класс ответы годовая контрольная работа по физике класс ответы административная контрольная работа по физике класс ответы итоговая контрольная работа по физике класс След Войти Версия Поиска Мобильная Полная Конфиденциальность Условия Настройки Отзыв Справка

ответы на контрольную работу по физике 8 класс вариант 3

AP Physics 1 Экзамен — AP Central

Информация об экзамене 2021
AP Physics 1 Экзамены будут предлагаться в бумажном виде в начале мая и в конце мая, а в виде цифрового экзамена — в начале июня. Как бумажная, так и цифровая версии экзамена AP Physics 1 будут иметь полную длину и будут измерять весь спектр навыков и знаний, указанных в описании курса и экзамена (CED), что даст студентам возможность получить зачетные баллы и размещение.

Бумажные экзамены будут проходить в стандартном формате с множественным выбором и свободным ответом, указанном в CED.Цифровой экзамен займет столько же времени, но будет содержать больше вопросов с несколькими вариантами ответов и меньше вопросов с бесплатными ответами. На цифровом экзамене студенты будут отвечать на вопросы с бесплатными ответами с клавиатуры, а не вручную. Приложение для цифрового экзамена будет содержать любые символы, которые понадобятся учащимся для ввода ответов. Студентов, сдающих цифровые экзамены, не попросят рисовать или рисовать в качестве части их ответа — скорее, эти навыки можно оценить, попросив учащихся дать объяснения по данным диаграмм или другим стимулам.Рукописные или сфотографированные работы не принимаются.

Педагоги и студенты смогут попрактиковаться в ответах на репрезентативную выборку вопросов с множественным выбором и свободного ответа в приложении для цифрового экзамена 2021 года, которое будет доступно в начале апреля. Лучшей практикой для цифровой версии экзамена AP Physics 1 2021 года остается вопрос с множественным выбором и свободным ответом.

Существуют различия между бумажной и цифровой версиями некоторых экзаменов AP 2021 года, и эти варианты различаются в зависимости от курса.Приведенная ниже информация относится только к AP Physics 1. Просмотрите сводку всех форматов экзаменов AP 2021.

Узнайте больше о тестировании 2021 года.

Обновление покрытия экзаменов
Примечание: Начиная с экзамена 2021 года, блоки 8–10 больше не будут тестироваться в AP Physics 1. Блоки 1-7 будут представлены на экзамене AP Physics 1 примерно в той же пропорции, что и их относительный вес, как указано в описании курса и экзамена.

AP Daily и AP Classroom
Короткие видеоролики AP Daily с возможностью поиска могут быть назначены вместе с тематическими вопросами, чтобы помочь вам охватить все содержание курса, навыки и модели задач, а также проверить понимание студентами.Разблокируйте личные проверки успеваемости, чтобы учащиеся могли продемонстрировать свои знания и навыки шаг за шагом, а также использовать панель управления прогрессом для выделения своего прогресса и дополнительных областей для поддержки. По мере приближения экзамена назначьте практические экзамены AP в банке вопросов AP Classroom и предложите студентам воспользоваться преимуществами онлайн-сессий с 19 по 29 апреля.

Войти в AP Classroom

Продвинутый практический тест по физике, ответы 14-15-16.pdf

Mt.Академия Эвереста Продвинутый Физика Практика Тест Ответы — Месяц 6, главы 14 , 15 , 16 понятий / словарный запас — 61 слово — по 1 баллу — возможен 61 балл 1) ___Periodic Motion________ — движение назад и вперед от положения равновесия 2) _Simple Harmonic Motion____ — (SHM ) — сила, возвращающая объект в положение равновесия, прямо пропорциональна смещению объекта. 3) _Period____________ (T) — Время, необходимое для завершения одного цикла движения 4) _Закон Гука __________ — сила, действующая на пружину, прямо пропорциональна величине растяжения пружины. F = -kx k — жесткость пружины [Н / м] 5) __Потенциальная энергия ____________- (в пружине) — PE sp = ½ kx 2 6) __Резонанс ____________ — когда небольшие силы прикладываются через равные промежутки времени к вибрирующему или колеблющемуся объекту и амплитуда вибрации увеличивается. 7) ____ Волна ____________ — возмущение, переносящее энергию через материю или пространство.8) _Wave pulse_________ — одиночный удар или возмущение, которое проходит через среду. 9) _Periodic wave________ — если волна движется вверх и вниз с одинаковой скоростью. 10) _Transverse wave______ — волна, которая колеблется перпендикулярно направлению движения волны. (например, струнные инструменты) 11) _Продольная волна_____ — волна, которая колеблется в направлении, параллельном направлению движения волны. (например, звук) 12) _Скорость волны _________ (υ) — смещение пика волны: Δd / Δt 13) ___ амплитуда ____________ (A) — расстояние между положением равновесия и максимальным смещением от равновесия. 14 ) __ Длина волны __________ (λ) — расстояние между любыми двумя гребнями или двумя впадинами. λ = v / f = vT 15 ) __Period___ (T) и ___Frequency________ (f) — время для завершения одного цикла (с) и циклов в секунду (Гц или с -1) f = 1 / T и T = 1 / f 16) Поведение волн зависит от их среды. На границах между средами волны могут переходить в новую среду или отражаться обратно в старую среду, либо и то, и другое. • Волна, которая достигает границы, называется волной ____incident_______.• Волна, которая возвращается от границы, — это __отраженная _____ волна. 17) _Принцип суперпозиции ____ — смещение среды, вызванное двумя или более волнами, представляет собой алгебраическую сумму смещений отдельных волн. Интерференция — результат наложения двух и более волн. 1

Советы по прохождению теста по предмету SAT Тест по физике

Для успешной сдачи предметного теста SAT по физике необходимы две вещи: глубокое знание материала, охватываемого тестом, и знание того, как сдавать тест. Поскольку экзамен слишком длинный, чтобы его могли сдать даже ученые-физики средней школы, для того, чтобы хорошо сдать экзамен, важно знать, как подойти к нему, чтобы максимально сэкономить время и получить максимальные баллы. Здесь я расскажу о наиболее важных стратегиях сдачи тестов, которые позволят вам хорошо сдать предметный тест SAT по физике.

1. Не задавайте вопросы в заказе

Вопросы на SAT по физике не отсортированы по степени сложности. То есть простые вопросы не в начале экзамена, а сложные — не в конце.Однако, если вы зададите вопросы по порядку, в конце могут возникнуть простые вопросы, которые вы полностью пропустите, потому что у вас закончилось время!

Чтобы избежать этого, отвечайте на вопросы в соответствии с вашим личным порядком сложности. Когда вы получите буклет (или когда будете сдавать пробные экзамены), сначала быстро просмотрите все вопросы и сразу же ответьте на те, которые очень легки для вас. Затем сделайте второй проход через экзамен, ответив на вопросы, требующие немного больше времени. Эта стратегия гарантирует, что вы действительно ответите на каждый вопрос, на который вы, , можете ответить, .

Персональный порядок сложности будет сильно различаться от человека к человеку, и вам нужно определить свои собственные трудности, пройдя диагностический тест. Например, один ученик может быть самым сильным в механике, потому что это последний курс, который он посещал. Другой студент, возможно, вообще не сможет ответить ни на какие задачи по механике, но сможет ответить на все вопросы по электромагнетизму.Другими словами: уровень сложности зависит от конкретного тестируемого, а не от какого-то универсального стандарта.

Пройдя диагностический тест, вы сможете определить свои сильные и слабые стороны. Это также одна из областей, где наставник может помочь. Репетиторы обучены находить, где ученикам нужна помощь, и направлять их, чтобы они могли повысить свои навыки в этих областях.

2. Если два ответа эквивалентны, оба неверны

Есть только один правильный ответ на любую задачу на Physics SAT. Используйте это в своих интересах! Если вы наткнулись на вопрос, в котором два варианта ответа эквивалентны, не стесняйтесь исключать их оба, поскольку оба они не могут быть правильными. Например, рассмотрим следующую задачу:

Эта задача просит вас определить, какая ось длины кажется изменяющейся, когда два объекта движутся со скоростью, близкой к скорости света. В этой задаче не столько важны части тела робота, сколько оси, на которых они существуют. Поставьте себя на место одного из роботов (неважно, какого именно).Если движение со скоростью, близкой к скорости света, заставит вас увидеть другого робота как более высокого, не изменится ли длина ног другого робота? Это означает, что оба эти ответа неверны. Точно так же, если движение таким образом изменит ширину тела робота (включая плечи), не изменится ли ширина лиц роботов? Это означает, что эти два варианта ответа также неверны! Последний вариант (B) — единственный, у которого нет эквивалента среди вариантов ответа, поэтому он должен быть правильным.

3. Знайте, когда нужно пропустить

В отличие от обычного SAT, в предметных тестах SAT все еще существует штраф за угадывание. В частности, на экзамене по физике за неправильный ответ дается -1/4 балла, а за отсутствие ответа — 0 баллов — ни прибавления, ни отнимания. Кроме того, экзамен слишком длинный, чтобы кто-либо мог ответить на все вопросы, и, если вы дадите все остальные ответы правильно, вы можете оставить до пятнадцати вопросов без ответа и все равно получить высший балл! Разработка стратегии пропуска вместо угадывания в ваших интересах не только для экономии времени, но и для увеличения вашего результата.Итак: когда угадывать, а когда пропускать?

В качестве упражнения чисто для оценки вероятностей рассмотрите свой средний выигрыш от угадывания, когда у вас есть 5,4,3 или 2 варианта ответа на выбор. Если у вас есть пять вариантов ответа, есть четыре неправильных, которые приведут к штрафу в размере -1/4 каждый, и один правильный ответ, который приведет к получению вами очков +1. Таким образом, в среднем количество очков, которое вы получите, если каждый раз будете угадывать, будет:

(4) * (- 1/4) + (1) * (1) = 0

Никакой пользы не будет, поэтому не в вашу пользу угадывать каждый вопрос.А теперь представьте, что вы можете исключить один определенно неправильный ответ и угадать среди остальных. Теперь у вас всего три неправильных ответа и один неправильный ответ:

(3) * (- 1/4) + (1) * (1) = 0,25

Теперь средний прирост положительный, и угадывание дает (среднее) преимущество! Точно так же, если вы можете исключить два неправильных ответа, средний выигрыш составит 0,5, а если вы исключите три, средний выигрыш составит 0,75.

Вы можете быть или не быть игроком.0,25 среднего выигрыша может быть недостаточно, чтобы соблазнить вас. Но общее правило состоит в том, что вы не должны гадать, если вы не можете исключить ни одного ответа. Оттуда все зависит от вас.

4. Если часть ответа неверна, значит и весь ответ неверен.

Процесс исключения — ваш лучший друг на этом экзамене. Если часть варианта ответа неверна, вы можете полностью исключить этот вариант ответа. Например, рассмотрим следующую задачу:

Для этой задачи вам нужно помнить, что скорость спутника на орбите вокруг массы M, расстояние r от центра массы равно

где G — гравитационная постоянная.Оба спутника вращаются на орбите с одинаковой массой, поэтому M одинаково для расчета для обоих спутников. Поскольку скорость обратно пропорциональна расстоянию, мы знаем, что тот, что дальше, будет медленнее. Мы еще не занимались математикой и уже можем исключить варианты ответов C и D! Отсюда он подключает 2r и r к уравнению и видит, что спутник 1 будет быстрее с коэффициентом корня 2 (A).

5. Пузырьки в ответах в группах

Это может показаться довольно глупым, но перенос ответов из буклета в лист для ответов после каждого вопроса занимает много времени.Вы намного быстрее передаете ответы, когда делаете несколько ответов за раз. По мере прохождения теста обведите варианты ответов в буклете. Затем, когда вы ответите 10–15 подряд, перенесите их все вместе в лист для ответов. Эта стратегия может сэкономить вам до двух минут на экзамене. Если у вас меньше минуты на вопрос, это дополнительное время очень дорого!

Если вам нужны дополнительные стратегии и советы о том, как подготовиться и сдать предметный тест SAT по физике, обратитесь к репетитору Cambridge Coaching сегодня!

Подробнее о физике ниже!

Список файлов

ИНСТРУКЦИИ (1.Щелкните курс Study Set, который вы хотите изучить.) (2. При желании вы можете нажать «Печать» и распечатать тестовую страницу (включая ответы, если вы щелкнули «Проверить ответы»). (3. Когда вы хотите Чтобы пройти тест … нажмите на любой из тестов для этого набора исследований.) (4. Нажмите «Проверить ответы», и он оценит ваш тест и исправит ваши ответы.) (5. Вы можете пройти все тесты как сколько раз вы выбираете, пока не получите «пятерку»!) (6. Автоматизированные курсы колледжа, созданные из конспектов лекций, классных экзаменов, учебников, материалов для чтения из многих колледжей и университетов.)
Это бесплатная услуга, предоставляемая компанией Why Fund Inc. (некоммерческая организация 501 C3). Мы благодарим вас за ваше пожертвование!
Информация о курсе
4361 Abeka Physics Глава 1 НАБОР ДЛЯ ИССЛЕДОВАНИЯ.htm
4362 Тест Abeka Physics Глава 1 MATCHING.htm
4363 Тест Abeka Physics Условия главы 1 МНОЖЕСТВЕННЫЙ ВЫБОР.htm
4364 Испытайте Abeka Physics термины главы 1 ИСТИНА ИЛИ ЛОЖЬ.htm
4365 Abeka Physics Глава 10 НАБОР ДЛЯ ИЗУЧЕНИЯ. htm
4366 Тест Abeka Physics Глава 10 MATCHING.htm
4367 Тест Abeka Physics Глава 10 НЕСКОЛЬКИЙ ВЫБОР.htm
4368 Тест Abeka Physics Глава 10 ИСТИНА ИЛИ ЛОЖЬ.htm
4369 Abeka Physics Глава 11 НАБОР ДЛЯ ИЗУЧЕНИЯ.htm
4370 Тест Abeka Physics Глава 11 MATCHING.htm
4371 Тест Abeka Physics Глава 11 НЕСКОЛЬКИЙ ВЫБОР.htm
4372 Тест Abeka Physics Глава 11 ИСТИНА ИЛИ ЛОЖЬ.htm
4373 Abeka Physics Глава 12 НАБОР ДЛЯ ИССЛЕДОВАНИЯ.htm
4374 Тест Abeka Physics Глава 12 MATCHING.htm
4375 Тест Abeka Physics Глава 12 НЕСКОЛЬКИЙ ВЫБОР. htm
4376 Тест Abeka Physics Глава 12 ИСТИНА ИЛИ ЛОЖЬ.htm
4377 Abeka Physics Глава 13 НАБОР ДЛЯ ИЗУЧЕНИЯ.htm
4378 Тест Abeka Physics Глава 13 МАТЧИРОВАНИЕ.htm
4379 Тест Abeka Physics Глава 13 НЕСКОЛЬКИЙ ВЫБОР.htm
4380 Проверьте Abeka Physics Глава 13 ИСТИНА ИЛИ ЛОЖЬ.htm
4381 Abeka Physics Глава 14 НАБОР ДЛЯ ИЗУЧЕНИЯ.htm
4382 Тест Abeka Physics Глава 14 MATCHING.htm
4383 Тест Abeka Physics Глава 14 МНОЖЕСТВЕННЫЙ ВЫБОР.htm
4384 Тест Abeka Physics Глава 14 ИСТИНА ИЛИ ЛОЖЬ.htm
4385 Abeka Physics Глава 15 НАБОР ДЛЯ ИЗУЧЕНИЯ. htm
4386 Тест Abeka Physics Глава 15 MATCHING.htm
4387 Тест Abeka Physics Глава 15 НЕСКОЛЬКИЙ ВЫБОР.htm
4388 Тест Abeka Physics Глава 15 ВЕРНО ИЛИ НЕВЕРНО.htm
4389 Abeka Physics Глава 16 НАБОР ДЛЯ ИЗУЧЕНИЯ.htm
4390 Тест Abeka Physics Глава 16 MATCHING.htm
4391 Test Abeka Physics Глава 16 НЕСКОЛЬКИЙ ВЫБОР.htm
4392 Test Abeka Physics Глава 16 ИСТИНА ИЛИ ЛОЖЬ.htm
4393 Abeka Physics Глава 17 НАБОР ДЛЯ ИССЛЕДОВАНИЯ.htm
4394 Тест Abeka Physics Глава 17 MATCHING.htm
4395 Тест Abeka Physics Глава 17 НЕСКОЛЬКИЙ ВЫБОР. htm
4396 Тест Abeka Physics Глава 17 ИСТИНА ИЛИ ЛОЖЬ.htm
4397 Abeka Physics Глава 18 НАБОР ДЛЯ ИЗУЧЕНИЯ.htm
4398 Тест Abeka Physics Глава 18 МАТЧИРОВАНИЕ.htm
4399 Тест Abeka Physics Глава 18 НЕСКОЛЬКИЙ ВЫБОР.htm
4400 Тест Abeka Physics Глава 18 ИСТИНА ИЛИ ЛОЖЬ.htm
4401 Abeka Physics Глава 19 НАБОР ДЛЯ ИЗУЧЕНИЯ.htm
4402 Тест Abeka Physics Глава 19 MATCHING.htm
4403 Тест Abeka Physics Глава 19 МНОЖЕСТВЕННЫЙ ВЫБОР.htm
4404 Test Abeka Physics Глава 19 ИСТИНА ИЛИ ЛОЖЬ.htm
4405 Abeka Physics Глава 2 НАБОР ДЛЯ ИССЛЕДОВАНИЯ. htm
4406 Тест Abeka Physics Глава 2 MATCHING.htm
4407 Тест Abeka Physics Глава 2 НЕСКОЛЬКИЙ ВЫБОР.htm
4408 Тест Abeka Physics Глава 2 ИСТИНА ИЛИ ЛОЖЬ.htm
4409 Abeka Physics Глава 20 НАБОР ДЛЯ ИЗУЧЕНИЯ.htm
4410 Тест Abeka Physics Глава 20 MATCHING.htm
4411 Тест Abeka Physics Глава 20 НЕСКОЛЬКИЙ ВЫБОР.htm
4412 Тест Abeka Physics Глава 20 ИСТИНА ИЛИ ЛОЖЬ.htm
4413 Abeka Physics Глава 21 НАБОР ДЛЯ ИССЛЕДОВАНИЯ.htm
4414 Тест Abeka Physics Глава 21 MATCHING.htm
4415 Тест Abeka Physics Глава 21 НЕСКОЛЬКИЙ ВЫБОР. htm
4416 Тест Abeka Physics Глава 21 ИСТИНА ИЛИ ЛОЖЬ.htm
4417 Abeka Physics глава 22 НАБОР ИЗУЧЕНИЯ.htm
4418 Тест Abeka Physics глава 22 СОПРЯЖЕНИЕ.htm
4419 Испытайте Abeka Physics, глава 22 НЕСКОЛЬКИЙ ВЫБОР.htm
4420 Test Abeka Physics глава 22 ПРАВДА ИЛИ ЛОЖЬ.htm
4421 Abeka Physics Глава 23 НАБОР ДЛЯ ИЗУЧЕНИЯ.htm
4422 Тест Abeka Physics Глава 23 MATCHING.htm
4423 Тест Abeka Physics Глава 23 МНОЖЕСТВЕННЫЙ ВЫБОР.htm
4424 Test Abeka Physics Глава 23 ИСТИНА ИЛИ ЛОЖЬ.htm
4425 Abeka Physics Глава 24 НАБОР ДЛЯ ИЗУЧЕНИЯ. htm
4426 Тест Abeka Physics Глава 24 MATCHING.htm
4427 Test Abeka Physics Глава 24 НЕСКОЛЬКИЙ ВЫБОР.htm
4428 Test Abeka Physics Глава 24 ИСТИНА ИЛИ ЛОЖЬ.htm
4429 Abeka Physics Глава 25 НАБОР ДЛЯ ИЗУЧЕНИЯ.htm
4430 Тест Abeka Physics Глава 25 MATCHING.htm
4431 Тест Abeka Physics Глава 25 НЕСКОЛЬКИЙ ВЫБОР.htm
4432 Test Abeka Physics Глава 25 ИСТИНА ИЛИ ЛОЖЬ.htm
4433 Abeka Physics Глава 26 НАБОР ДЛЯ ИССЛЕДОВАНИЯ.htm
4434 Тест Abeka Physics Глава 26 MATCHING.htm
4435 Тест Abeka Physics Глава 26 НЕСКОЛЬКИЙ ВЫБОР. htm
4436 Тест Abeka Physics Глава 26 ИСТИНА ИЛИ ЛОЖЬ.htm
4437 Abeka Physics Глава 27 НАБОР ДЛЯ ИЗУЧЕНИЯ.htm
4438 Тест Abeka Physics Глава 27 МАТЧИРОВАНИЕ.htm
4439 Тест Abeka Physics Глава 27 НЕСКОЛЬКИЙ ВЫБОР.htm
4440 Test Abeka Physics Глава 27 ИСТИНА ИЛИ ЛОЖЬ.htm
4441 Abeka Physics Глава 28 НАБОР ДЛЯ ИЗУЧЕНИЯ.htm
4442 Тест Abeka Physics Глава 28 MATCHING.htm
4443 Тест Abeka Physics Глава 28 МНОЖЕСТВЕННЫЙ ВЫБОР.htm
4444 Test Abeka Physics Глава 28 ИСТИНА ИЛИ ЛОЖЬ.htm
4445 Abeka Physics Глава 3 НАБОР ДЛЯ ИССЛЕДОВАНИЙ. htm
4446 Тест Abeka Physics Глава 3 MATCHING.htm
4447 Тест Abeka Physics Глава 3 НЕСКОЛЬКИЙ ВЫБОР.htm
4448 Тест Abeka Physics Глава 3 ВЕРНО ИЛИ НЕВЕРНО.htm
4449 Abeka Physics Глава 4 НАБОР ДЛЯ ИССЛЕДОВАНИЙ.htm
4450 Тест Abeka Physics Глава 4 MATCHING.htm
4451 Тест Abeka Physics Глава 4 НЕСКОЛЬКИЙ ВЫБОР.htm
4452 Тест Abeka Physics Глава 4 ИСТИНА ИЛИ ЛОЖЬ.htm
4453 Abeka Physics Глава 5 НАБОР ДЛЯ ИССЛЕДОВАНИЯ.htm
4454 Тест Abeka Physics Глава 5 MATCHING.htm
4455 Тест Abeka Physics Глава 5 НЕСКОЛЬКИЙ ВЫБОР. htm
4456 Тест Abeka Physics Глава 5 ИСТИНА ИЛИ ЛОЖЬ.htm
4457 Abeka Physics Глава 6 НАБОР ДЛЯ ИЗУЧЕНИЯ.htm
4458 Тест Abeka Physics Глава 6 СОГЛАСОВАНИЕ.htm
4459 Испытайте Abeka Physics Глава 6 НЕСКОЛЬКИЙ ВЫБОР.htm
4460 Тест Abeka Physics Глава 6 ИСТИНА ИЛИ ЛОЖЬ.htm
4461 Abeka Physics Глава 8 НАБОР ДЛЯ ИЗУЧЕНИЯ.htm
4462 Тест Abeka Physics Глава 8 MATCHING.htm
4463 Тест Abeka Physics Глава 8 МНОЖЕСТВЕННЫЙ ВЫБОР.htm
4464 Тест Abeka Physics Глава 8 ИСТИНА ИЛИ ЛОЖЬ.htm
4465 Abeka Physics Глава 9 НАБОР ДЛЯ ИЗУЧЕНИЯ. htm
4466 Тест Abeka Physics Глава 9 MATCHING.htm
4467 Тест Abeka Physics Глава 9 НЕСКОЛЬКИЙ ВЫБОР.htm
4468 Тест Abeka Physics Глава 9 ИСТИНА ИЛИ ЛОЖЬ.htm
4469 Финальный экзамен Honor Physics STUDY SET.htm
4470 Заключительный экзамен Test Honor по физике MATCHING.htm
4471 Финальный экзамен Test Honor по физике MULTIPLE CHOICE.htm
4472 Финальный экзамен Test Honor по физике ИСТИНА ИЛИ ЛОЖЬ.htm
4473 Награды по физике Глава 1 УЧЕБНЫЙ НАБОР.htm
4474 Тест с отличием по физике Глава 1 MATCHING.htm
4475 Test Honors Physics Chapter 1 MULTIPLE CHOICE. htm
4476 Test Honors Physics Chapter 1 TRUE OR FALSE.htm
4477 Награды Physics Chapter 10 STUDY SET.htm
4478 Тест с отличием по физике Глава 10 МАТЧИРОВАНИЕ.htm
4479 Test Honors Physics Chapter 10 MULTIPLE CHOICE.htm
4480 Test Honors Physics Chapter 10 ИСТИНА ИЛИ ЛОЖЬ.htm
4481 отмечает главу 11 по физике STUDY SET.htm
4482 Глава 11 по физике: MATCHING.htm
4483 Тест посвящен главе 11 по физике МНОЖЕСТВЕННЫЙ ВЫБОР.htm
4484 Тест посвящен главе 11 по физике ИСТИНА ИЛИ ЛОЖЬ.htm
4485 Награды по физике Глава 12 STUDY SET. htm
4486 Тест с отличием по физике Глава 12 MATCHING.htm
4487 Тест с отличием по физике Глава 12 НЕСКОЛЬКИЙ ВЫБОР.htm
4488 Test Honors Physics Глава 12 ИСТИНА ИЛИ ЛОЖЬ.htm
4489 Награды Physics Chapter 13 STUDY SET.htm
4490 Тест с отличием по физике Глава 13 MATCHING.htm
4491 Тест с отличием по физике Глава 13 НЕСКОЛЬКИЙ ВЫБОР.htm
4492 Test Honors Physics Глава 13 ИСТИНА ИЛИ ЛОЖЬ.htm
4493 Награды по физике Глава 14 НАБОР ИЗУЧЕНИЯ.htm
4494 Тест с отличием по физике Глава 14 MATCHING.htm
4495 Test Honors Physics Chapter 14 MULTIPLE CHOICE. htm
4496 Test Honors Physics Глава 14 ИСТИНА ИЛИ ЛОЖЬ.htm
4497 Награды Physics Chapter 15 STUDY SET.htm
4498 Тест с отличием по физике Глава 15 МАТЧИРОВАНИЕ.htm
4499 Тест с отличием по физике Глава 15 НЕСКОЛЬКИЙ ВЫБОР.htm
4500 Test Honors Physics Глава 15 ИСТИНА ИЛИ ЛОЖЬ.htm
4501 Награды Physics Chapter 16 STUDY SET.htm
4502 Тест с отличием по физике Глава 16 MATCHING.htm
4503 Тест с отличием по физике Глава 16 НЕСКОЛЬКИЙ ВЫБОР.htm
4504 Test Honors Physics Глава 16 ИСТИНА ИЛИ ЛОЖЬ.htm
4505 Награды за физику Chapter 17-18 STUDY SET. htm
4506 Test Honors Physics Глава 17-18 MATCHING.htm
4507 Test Honors Physics Глава 17 — 18 МНОЖЕСТВЕННЫЙ ВЫБОР.htm
4508 Test Honors Physics Глава 17-18 TRUE OR FALSE.htm
4509 Награды по физике Глава 17 STUDY SET.htm
4510 Test Honors Physics Глава 17 MATCHING.htm
4511 Test Honors Physics Chapter 17 MULTIPLE CHOICE.htm
4512 Test Honors Physics Глава 17 ИСТИНА ИЛИ ЛОЖЬ.htm
4513 Награды Physics Chapter 19 STUDY SET.htm
4514 Тест с отличием по физике Глава 19 MATCHING.htm
4515 Test Honors Physics Chapter 19 MULTIPLE CHOICE. htm
4516 Test Honors Physics Глава 19 ИСТИНА ИЛИ ЛОЖЬ.htm
4517 С отличием по физике Глава 2 УЧЕБНЫЙ НАБОР.htm
4518 Test Honors Physics Chapter 2 MATCHING.htm
4519 Test Honors Physics Chapter 2 MULTIPLE CHOICE.htm
4520 Test Honors Physics Глава 2 ИСТИНА ИЛИ ЛОЖЬ.htm
4521 Награды по физике Глава 20 STUDY SET.htm
4522 Тест с отличием по физике Глава 20 МАТЧИРОВАНИЕ.htm
4523 Test Honors Physics Chapter 20 MULTIPLE CHOICE.htm
4524 Test Honors Physics Глава 20 ИСТИНА ИЛИ ЛОЖЬ.htm
4525 Награды по физике — Глава 21-22 STUDY SET. htm
4526 Тест с отличием по физике, главы 21 и 22 МАТЧИРОВАНИЕ.htm
4527 Test Honors Physics Глава 21 и 22 MULTIPLE CHOICE.htm
4528 Тест с отличием по физике, главы 21 и 22 ИСТИНА ИЛИ ЛОЖЬ.htm
4529 Награды по физике Глава 3 STUDY SET.htm
4530 Тест с отличием по физике Глава 3 МАТЧИРОВАНИЕ.htm
4531 Тест с отличием по физике Глава 3 НЕСКОЛЬКИЙ ВЫБОР.htm
4532 Test Honors Physics Chapter 3 ИСТИНА ИЛИ ЛОЖЬ.htm
4533 Награды Physics Chapter 4 STUDY SET.htm
4534 Тест с отличием по физике Глава 4 MATCHING.htm
4535 Тест с отличием по физике Глава 4 НЕСКОЛЬКИЙ ВЫБОР. htm
4536 Test Honors Physics Глава 4 ИСТИНА ИЛИ ЛОЖЬ.htm
4537 Награды по физике Глава 5 STUDY SET.htm
4538 Тест с отличием по физике Глава 5 MATCHING.htm
4539 Тест с отличием по физике Глава 5 НЕСКОЛЬКИЙ ВЫБОР.htm
4540 Тест с отличием по физике Глава 5 ИСТИНА ИЛИ ЛОЖЬ.htm
4541 Награды по физике Глава 6 STUDY SET.htm
4542 Тест с отличием по физике Глава 6 MATCHING.htm
4543 Test Honors Physics Chapter 6 MULTIPLE CHOICE.htm
4544 Test Honors Physics Глава 6 ИСТИНА ИЛИ ЛОЖЬ.htm
4545 Награды по физике Глава 7 УЧЕБНЫЙ НАБОР. htm
4546 Тест с отличием по физике Глава 7 MATCHING.htm
4547 Test Honors Physics Chapter 7 MULTIPLE CHOICE.htm
4548 Test Honors Physics Глава 7 ИСТИНА ИЛИ ЛОЖЬ.htm
4549 С отличием по физике Глава 8 STUDY SET.htm
4550 Тест с отличием по физике Глава 8 МАТЧИРОВАНИЕ.htm
4551 Test Honors Physics Chapter 8 MULTIPLE CHOICE.htm
4552 Test Honors Physics Глава 8 ИСТИНА ИЛИ ЛОЖЬ.htm
4553 Награды по физике Глава 9 STUDY SET.htm
4554 Тест с отличием по физике Глава 9 MATCHING.htm
4555 Тест с отличием по физике Глава 9 НЕСКОЛЬКИЙ ВЫБОР. htm
4556 Test Honors Physics Глава 9 ИСТИНА ИЛИ ЛОЖЬ.htm
4557 Награды Physics Chapters 20-23 STUDY SET.htm
4558 Test Honors Physics Chapters 20-23 MATCHING.htm
4559 Test Honors Physics Chapters 20-23 MULTIPLE CHOICE.htm
4560 Тест с отличием по физике, главы 20-23 ВЕРНО ИЛИ НЕВЕРНО.htm
4561 Награды: Physics Test 1 STUDY SET.htm
4562 Тест с отличием по физике Тест 1 MATCHING.htm
4563 Тест с отличием по физике Тест 1 MULTIPLE CHOICE.htm
4564 Test Honors Physics Test 1 TRUE OR FALSE.htm
4565 Тест по физике с отличием 2 НАБОР ИЗУЧЕНИЯ. htm
4566 Test Honors Physics Test 2 MATCHING.htm
4567 Test Honors Physics Test 2 MULTIPLE CHOICE.htm
4568 Test Honors Physics Test 2 TRUE OR FALSE.htm
4569 Награды за Physics Test 3 STUDY SET.htm
4570 Тест с отличием по физике Тест 3 МАТЧИРОВАНИЕ.htm
4571 Test Honors Physics Test 3 MULTIPLE CHOICE.htm
4572 Test Honors Physics Test 3 ИСТИНА ИЛИ ЛОЖЬ.htm
4573 Важные физические формулы для MCAT STUDY SET.htm

Физика, с отличием — Физика

CH 14-15 презентации Power Point

2017-2018

Экзамен Ch 5 вторник, 21 ноября.

Канал 5 Рекомендуемая практика:

Вопросы 1, 5, 6, 10, 12, 15

Вопросы: 3, 5, 6, 8, 10, 12, 15, 17, 19, 22, 25, 27, 33, 34, 37, 42

Экзамен Ch 4, четверг, 9 ноября.

первая викторина на канале 4, пятница, 20 октября.

13.10.17

Четвертая глава очень важна. Представляя законы движения Ньютона, он также представляет концепцию FBD диаграммы свободного тела, которая не только помогает студенту в решении задач по физике, но и требуется на тестах и ​​викторинах.

Гл. 4 предлагает практику из 3-го издания. Проблемы и вопросы доступны по ссылке ниже:

ch 4 Q’s: 2, 4, 6, 8, 10, 14, 117, 18, 23, 24

ch 4 P’s: 2, 3, 4/11 , 14, 16/19, 21, 22, 24, 26, 30, 32/36, 39, 40, 42, 46/51, 53, 55, 60, 62/70, 74, 75, 81, 88, 108

Тест по главе 2-3, четверг, 5 октября 2017 года.

Лабораторное упражнение по определению g.

Главы 2 и 3:

Одномерная и двумерная кинематика (изучение движения без использования концепции силы) рассматривается во второй и третьей главах. Будет проведено как минимум две викторины, за которыми последует тест, охватывающий обе главы. Я также покажу формат проблемы, который, как я ожидаю, будет соблюдаться для всех представленных решений проблем.

CH 2-3 Рекомендуемая практика

ch 2 Q’s: 1, 4, 5, 6, 8, 9, 12

ch 2 P: 3, 8/13, 15, 17/19, 21, 23, 29, 32/40, 41, 42, 46, 51, 55/59, 60, 61, 65

ch 3 q’s: 1, 2, 3, 7, 8, 11, 16

ch 3 P: 7, 9, 11/14, 15, 18, 26, 28, 29, 31, 36


Lab Matching lab

Как вам там хорошо известно много веб-сайтов, которые помогут вам понимать концепции и помогать решать проблемы.Я понимаю что ты можешь возможно, найду законченные решения домашних заданий и тестовых задач. Используйте их, чтобы помочь в вашем концептуальном понимании и способности решать проблемы. Я перечислил некоторые из них, которые, как мне кажется, могут помочь вам в нашей физике. класс.

Я буду доступен для помощи в понедельник после школы в течение как минимум 1/2 часа.

Ваш текст — Physics by Cutnell / Johnson Physics , 3-е или 5-е издание.

Оценки рассчитываются по общему количеству баллов. Тем не менее, ваши оценки за тест вносят наибольший вклад в вашу оценку.Викторины могут быть НЕОБЪЯВЛЕННЫХ !
и учитывайте примерно 25% вашей итоговой оценки. Нет составов викторины; но одна оценка викторины за семестр будет снижена. Планируйте соответственно.

Тесты и викторины включают вопросы с несколькими вариантами ответов и вопросы с открытым ответом. Примерно 80% вопросов теста с открытым ответом и викторины — это текст, никаких уловок. По ссылкам на этом веб-сайте можно найти практические вопросы с множественным выбором ответов. Большинство этих вопросов MC будут качественными.Прочтите свой текст, чтобы улучшить свое концептуальное понимание материала. Тесты обычно даются в конце каждой главы; хотя объявлено, вы должны знать, когда они придут, по нашим успехам в классе. Конец chapter = ТЕСТ. Тест рассчитан примерно на 65% вашей оценки. Если вы пропустите тест по уважительной причине, макияж будет в следующий понедельник после школы. Например, если вы пропустите тест во вторник, 19 сентября, дата / время подготовки для этого теста будут после школы, с 2:50 до 3:50, в понедельник 25 сентября.Опять же, ПЛАН Соответственно. Отчеты

должны быть сданы на следующий день. Используйте формат отчета и время, с которыми вы знакомы по HChem.

Я опубликует «предлагаемую практику». Проблемы разделены по главам и раздел; Как правило, мы будем следовать тексту, если не объявлено в классе. Вы несете ответственность за то, чтобы ваша учеба соответствовала моим темпам в классе. Уравнения и единицы не задаются и не публикуются во время викторин и экзаменов.

Я буду ставить оценки в конце каждого семестра , если Я чувствую это необходимо.Я дам средний класс каждого теста и викторина, если требуется. Отметьте со шкалой или без нее ваш срок и окончание оценки не станут сюрпризом. Если ваши результаты тестов выше среднего, ваша оценка будет выше среднего. Дополнительного кредита нет.

я Ожидаю, что вы будете работать над ФИЗИКОЙ, когда будете в моей комнате. Относитесь с уважением к другие и класс; молчите, если вы не вносите вклад в класса, содержите наш класс в чистоте, верните все учебные материалы в их надлежащее место.

2016-2017

Как вам хорошо известно, существует множество веб-сайтов, которые помогут вам понять концепции и помочь с проблемами.Я понимаю, что вы, вероятно, сможете найти готовые решения для своих домашних заданий и тестовых задач. Используйте их, чтобы помочь в вашем концептуальном понимании и способности решать проблемы. Я перечислил некоторые из них, которые, как мне кажется, могут помочь вам на нашем уроке физики.

Если у меня нет назначенных администрацией встреч, я буду доступен для помощи после школы в течение как минимум 1/2 часа.

Ваш текст — Физика Катнелла / Джонсона, 3-е или 5-е издание.

Примерно 80% вопросов тестов и викторин взяты из текста, без уловок; Если вы концептуально понимаете текстовый материал и имеете навыки решать проблемы в тексте, вы справитесь.

Я выложу «рекомендуемые практики». Проблемы разделены по главам и разделам; Как правило, мы будем следовать тексту, если не объявлено в классе. Вы несете ответственность за то, чтобы ваша учеба соответствовала моим темпам в классе. Уравнения и единицы не задаются и не публикуются во время викторин и экзаменов.

Викторины могут быть НЕОБЪЯВЛЕННЫМИ ! и рассчитывайте примерно на 25% вашей итоговой оценки.

Как правило, тесты приводятся в конце главы; хотя они объявлены, вы должны знать, когда они появятся, по нашим успехам в классе.Конец главы = ТЕСТ. Тест рассчитан примерно на 65% вашей оценки.

Я буду ставить оценки в конце каждого семестра , если считаю, что это необходимо. Я дам средний класс по каждому тесту и тесту, если потребуется. Отметьте, со шкалой или без нее, ваш семестр и итоговые оценки не станут сюрпризом. Если ваши результаты тестов выше среднего, ваша оценка будет выше среднего. НЕТ дополнительных кредитов.

Я ожидаю, что вы будете заниматься ФИЗИКОЙ, когда будете в моей комнате. Относитесь с уважением к другим и к классу; молчите, если вы не участвуете в уроке, содержите наш класс в чистоте, верните все учебные материалы в надлежащее место.

В первой главе рассматриваются единицы измерения, преобразование единиц измерения, простая тригонометрия и ее применение к векторам, а также сложение векторов. Я ожидаю, что это будет обзор для всех, кого рекомендуют для этого курса. Если вы не понимаете этого или другого материала в будущих главах, вам следует прочитать текст и попробовать предлагаемую практику. Вам будет сложно пройти этот курс без понимания этого материала.

Информация о солнечных батареях AHS

Главы 12-15

Обратите внимание, я ожидаю, что вы знаете, полученные на уроках химии (температурные шкалы, pv = nRT, Q = mc (Tf-T0)…), большая часть материала из этих глав. Я расскажу о материале, который, как мне кажется, является новым для студентов-физиков с отличием. ВЫ должны прочитать эти главы, изучить предлагаемые практические задачи, а затем быть готовыми задавать вопросы при необходимости.

CH 12 рекомендуемая практика: P’s 11, 18, 32, 42, 48, 60,

Ch 13 рекомендуемая практика:

2, 3, 7, 8, 15, 19, 21, 26, 38, 39

Раздел 14, рекомендуемая практика: вопросы 3, 5, 9, 13

Вопросы: 4, 7, 11, 13, 17, 19, 26, 37, 38, 48

СН 15 Рекомендуемая практика:

10, 12, 15 , 17, 20, 21, 22, 24, 30, 33, 36, 39, 43, 44, 46, 50, 51, 53, 64, 65

CH.11 Рекомендуемые практики

Вопросы: 4, 5, 7, 9, 11, 19, 22, 26

Задачи: 2, 6, 7, 11, 13, 21, 24, 27, 31, 34, 36, 40 , 42, 53, 54, 57, 59, 61, 64, 67

Вопросы: CH 10 концептуальных вопросов с ответами для вашего обзора

Проблемы: 1, 4, 5, 7, 10, 12, 23, 25, 30 , 36, 40, 48, 51, 52, 63, 65, 67

1/3/2016

Ch 9 Рекомендуемая практика по динамике вращения

Вопросы: 2, 6, 13, 18, 19, 23

Проблемы: 2, 4, 6, 9, 12, 15, 17, 20, 29, 32, 34, 39, 41, 45, 48, 53, 57, 58, 60

Глава 7 Impulse Momentum Рекомендуемая практика

Вопросы: 4 , 6, 8, 9, 11, 12, 16

Проблемы: 4, 5, 7, 8, 11, 15, 16, 18, 22, 25, 28, 29, 30, 32, 33, 37, 41, 44

Глава 6, рекомендуемая практика : (обратите внимание, что порядок задач скорректирован, чтобы отразить порядок разделов, рассматриваемых в классе)

Вопросы: 1, 6, 8, 10, 12, 13, 16

P: 3, 5, 7, 10, / 27, 28, 31/33, 35 , 39, 42, 43, 44/14, 15, 17, 18, 20, 24/47, 48, 51, 52, 54/58, 59, 60, 62, 64, / 69, 71

Канал 5 и вращательная кинематика Тест Четверг, 12 ноября

Раздел 5 Рекомендуемая практика:

Q’s 1, 5, 6, 10, 12, 15

P: 3, 5, 6, 8, 10, 12, 15, 17, 19 , 22, 25, 27, 33, 34, 37, 42

Канал 1 Рекомендуемая практика:

канал 1 Вопросы: 3, 4, 10, 11, 1
вп 1 П: 5, 10/11, 13, 18, 19/21, 23, 29/33/35, 39/42, 44, 46, 49, 51

Катнелл Джонсон 3-е изд. вопросы и проблемы

Вы можете пройти тест по этому материалу во вторник, 6 сентября.

Одномерная и двумерная кинематика (изучение движения без использования концепции силы) рассматривается во второй и третьей главах. Будет проведено как минимум две викторины, за которыми последует тест, охватывающий обе главы. Я также покажу формат проблемы, который, как я ожидаю, будет соблюдаться для всех представленных решений проблемы.

CH 2-3 Рекомендуемая практика

ch 2 Q’s: 1, 4, 5, 6, 8, 9, 12

ch 2 P: 3, 8/13, 15, 17/19, 21, 23, 29, 32/40, 41, 42, 46, 51, 55/59, 60, 61, 65

ch 3 q’s: 1, 2, 3, 7, 8, 11, 16

ch 3 P: 7, 9, 11/14, 15, 18, 26, 28, 29, 31, 36

Четвертая глава очень важна.Представляя законы движения Ньютона, он также представляет концепцию FBD диаграммы свободного тела, которая не только помогает студенту в решении задач по физике, но и требуется на тестах и ​​викторинах.

Ch 4 Рекомендуемая практика:

ch 4 Q’s: 2, 4, 6, 8, 10, 14, 117, 18, 23, 24

ch 4 P: 2, 3, 4/11, 14, 16 / 19, 21, 22, 24, 26, 30, 32/36, 39, 40, 42, 46/51, 53, 55, 60, 62/70, 74, 75, 81, 88, 108

сообщений от 2015 — 2016

Предлагаемые задачи: 5, 6, 11, 13. 21, 23, 27, 30, 32, 38, 43, 46, 49, 55, 58, 62


Тест по главе 11 Пятница 18.03.16


Рекомендуемая практика в главе 11:

Проблемы: 2, 7, 9, 11, 13, 15, 21, 23, 27, 28, 31, 34, 36, 38, 40, 43, 44, 53, 55, 57, 59, 62, 64, 68, 69

Глава 9 Предлагаемая практика по динамике вращения

Вопросы: 2, 6, 13, 18, 19, 23

Задачи: 2, 4, 6, 9, 12, 15, 17, 20, 29, 32, 34, 39 , 41, 45, 48, 53, 57, 58, 60

CH 8 Рекомендуемая практика вращательной кинематики

Вопросы: 3, 6, 9, 13

Задачи: 2, 6, 9, 14, 17, 20, 23 , 30, 33, 37, 43, 44, 49, 50, 54, 56

Глава 9 Предлагаемые практические задачи: 6, 8, 13, 15, 20, 30, 32, 41, 45, 47, 51, 53, 60


Оценка ротации Среда 3/2/16

Глава 7 Рекомендуемая практика:

задач 1, 3, 4, 6, 7, 11, 14, 15, 17, 20, 27, 32 , 35, 36, 38, 41, 43, 44

Глава 6 Рекомендуемая практика:

вопросов: 2, 4, 8, 12, 15, 17

задач: 5, 7, 9, 11, 14, 17, 22, 24, 28, 30 , 31, 33, 35, 40, 42, 44, 51, 56, 59, 62, 69, 70, 71

Рекомендуемая практика в главе 4:

вопросов: 2, 3, 5, 8, 1014 , 16, 18, 22, 26

задач: 5, 7, 11, 14, 16, 19, 21, 26, 28, 29, 32, 33, 38, 42, 43, 46, 52, 53, 55, 60, 61, 62, 70, 72, 75, 78, 88

Завершите практику по разделам, прежде чем глава 2 будет завершена, будет хотя бы одна, если не две необъявленные викторины.

Рекомендуемая практика в главе 2:

вопроса: 1, 3, 5, 9 (показать результат расчетом), 12

Задачи: 2, 6, 9, 13, 17, 19, 27, 29, 32, 37, 45, 48, 52, 53, 55, 59, 60, 61, 63

CH 15 Рекомендуемая практика:

10, 12, 15, 17, 20, 21, 22, 24, 30, 33, 36, 39, 43, 44, 46, 50, 51, 53, 64, 65

Глава 13 Рекомендуемая практика:

2, 3, 7, 8, 15, 19, 21, 26, 38, 39

Главы 12-15

Обратите внимание, я ожидаю, что вы узнаете, полученные на уроках химии (температурные шкалы, pv = nRT, Q = mc (Tf-T0)…), большая часть материала из этих глав. Я расскажу о материале, который, как мне кажется, является новым для студентов-физиков с отличием. ВЫ должны прочитать эти главы, изучить предлагаемые практические задачи, а затем быть готовыми задавать вопросы при необходимости.

СН. 11 Рекомендуемые практики

Вопросы: 4, 5, 7, 9, 11, 19, 22, 26

Задачи: 2, 6, 7, 11, 13, 21, 24, 27, 31, 34, 36, 40 , 42, 53, 54, 57, 59, 61, 64, 67

Ch 9 Предлагаемая практика по динамике вращения

Вопросы: 2, 6, 13, 18, 19, 23

Задачи: 2, 4, 6, 9 , 12, 15, 17, 20, 29, 32, 34, 39, 41, 45, 48, 53, 57, 58, 60

CH 8 Рекомендуемая практика с вращательной кинематикой

Вопросы: 3, 6, 9, 13

Задачи: 2, 6, 9, 14, 17, 20, 23, 30, 33, 37, 43, 44, 49, 50, 54, 56

Ch 7 Impulse Momentum Рекомендуемая практика

Вопросы: 4, 6, 8, 9, 12, 15, 16

Проблемы: 4, 5, 7, 8, 11, 15, 17, 18, 22, 25, 26, 28, 29, 30, 33, 36

Видео предложения по главе 6 :

http: // ocw. mit.edu/courses/physics/8-01-physics-i-classical-mechanics-fall-1999/video-lectures/lecture-11/

http://aplusphysics.com/courses/regents/videos/ConsEnergy/ ConsEnergy.html

http://aplusphysics.com/courses/regents/videos/Power/Power.html

Рекомендуемая практика в главе 6 : проблемы: (обратите внимание, что порядок проблем скорректирован с учетом порядка разделов покрыты в классе) 3, 5, 7, 10, / 27, 28, 31/33, 35, 39, 42, 43, 44/14, 15, 17, 18, 20, 23, 24/47, 48, 51 , 52, 54/58, 59, 60, 62, 64, / 69, 71

Прочтите / задайте вопросы / обратите внимание на уроке / попробуйте решать текстовые задачи!

Рекомендуемая практика в главе 5 : задачи: 3, 4, 5, 7, 8, 11, 15, 18, 21, 22, 25, 27, 31, 32, 34, 37, 40, 42. вопросы по проверке оформления заказа.

Викторина «Равновесие и динамика».

Главы 2–4 закладывают важную основу для всего курса: Читайте / задавайте вопросы / обращайте внимание на классе / практикуйте текстовые задачи!

Рекомендуемая практика / исследование

запомнить уравнения

опубликовано: 9/4/14. Глава 4: Прочитать при необходимости полностью. Задачи: 1, 3, 5/11, 15, 17, / 19, 23, 25, 29, 31, 33/39, 41, 43, 45, 47/49, 53, 55, 57, 60, 61, 63 / 70, 73, 75, 77, 79, 88/108

Кабинет физики on-line:

www. физический класс .com /

Giancoli Ch 2 1D кинематика

Test Your Gravity IQ — Физика для аниматоров

Журнал

CG Freelancer Magazine в сотрудничестве с Physics for Animators представил вам викторину Gravity IQ, чтобы проверить ваши основные знания о гравитации, применяемой в анимации. Если вы не проходили тест, вы можете сделать это здесь, прежде чем просматривать ответы ниже. Не подглядывать!


Как вы прошли викторину Gravity IQ?

  • 100 очков или больше : вы — мастер гравитации! Поклонитесь и похлопайте себя по спине.Вы готовы убедительно анимировать гравитационные эффекты.
  • 70–100 баллов : Вероятно, вы можете неплохо анимировать сцены с гравитацией, но вам лучше просмотреть ответы ниже, чтобы завершить свое образование.
  • 40–70 баллов: Вы не новичок, но вы могли бы пройти курс повышения квалификации, прежде чем пытаться анимировать с использованием гравитации.
  • 0-40 баллов : Вы можете передвигаться, не падая, но лучше не пытаться анимировать, пока не ознакомитесь с ответами ниже.


Правильные ответы

Вот и правильные ответы, и частично правильные.

1. Какие из следующих утверждений относительно гравитации на поверхности Земли верны?

  • Гравитация притягивает объекты к центру Земли. Сила тяжести тянется к центру Земли. Поэтому люди на другом конце Земли не падают!
  • Гравитация удерживает нашу атмосферу на месте. Наша атмосфера удерживается на Земле гравитацией. Нам повезло, иначе мы не смогли бы дышать.
  • Атмосферное давление на поверхности Земли выше, чем на высоте нескольких миль, потому что гравитация давит на атмосферу. Гравитация постоянно притягивает атмосферу к центру Земли. В рамках этой деятельности воздух на внешних границах атмосферы постоянно давит на воздух под ним, сжимая его. Давление, которое это создает на поверхности, — это наше повседневное «давление воздуха».По этой же причине давление воздуха ниже в воздухе высоко над Землей; там, вверху, на него не так много воздуха.
  • Гравитационное притяжение на экваторе немного отличается от гравитационного притяжения на Северном и Южном полюсе. Это правда, хотя это не очень хорошо известно. Вращение Земли, наблюдаемое на экваторе по сравнению с Северным или Южным полюсом, влияет на то, как гравитация ощущается в этих местах. На экваторе гравитационное притяжение немного меньше. Если вы это правильно поняли, вы действительно заядлый студент гравитации!

2.В классической физике гравитация равна…

  • сила (правильный ответ)
  • тип ускорения (частичные точки)
    В классической физике гравитация считается силой, а силы вызывают ускорение.

3. В каких единицах выражается ускорение свободного падения?

  • Выражается гравитационное ускорение То же, что и любое другое ускорение , как Расстояние за время за время .

4. Каково приблизительное ускорение свободного падения у поверхности Земли?

  • 9,8 м / сек / сек
  • 32 фута / сек / сек
    Если вы собираетесь анимировать падающих людей или предметы, вам нужно знать это холодно!

5. Гравитационное притяжение, которое планета (или луна) оказывает на свою поверхность, зависит от ее:

  • масса — Это правильный ответ.Масса связана с размером и плотностью.
  • размер — Частичные баллы даны за связанный ответ.
  • вес — Вес — это масса, на которую влияет сила тяжести, поэтому это связанный ответ (частичные баллы).
  • плотность — Частичные баллы даны за связанный ответ.
  • скорость вращения — На гравитационное притяжение очень мало влияет скорость вращения на экваторе планеты относительно ее полюсов (см. Вопрос 1). Частичные баллы.

6. Выберите наилучшее описание взаимосвязи между массой и весом.

  • Вес — это мера силы тяжести на массу.
    Не расстраивайтесь, если вы ошиблись. Это может сбить с толку, если вы не изучили разницу между массой и весом!

7. Предположим, вы стоите на крыше и в одно и то же время роняете апельсин и носовой платок. Апельсин первым ударится о землю, потому что:

  • Легкая ткань больше подвержена сопротивлению воздуха, которое замедляет падение платка больше, чем апельсина.
    На этот вопрос часто отвечают неправильно даже аниматоры, проработавшие в бизнесе десятилетиями. Сопротивление воздуха, возникающее в результате отталкивания воздуха от падающего объекта, отвечает за большинство «антигравитационных» эффектов, которые мы наблюдаем в жизни, таких как летающие птицы, самолеты и парашюты, замедляющие падение прыгуна. На каждый объект гравитация действует одинаково, но может казаться, что на него влияет по-разному из-за направленных вверх сил, таких как сопротивление воздуха.


Теперь, когда вы знаете, хотите попробовать еще раз? Пройди тест еще раз! И загрузите бесплатное приложение CG Freelancer Magazine на свое устройство iOS или Android.Следующий номер журнала CG Freelancer — о гравитации!

И если вы хотите поспорить со мной по поводу любого из этих ответов, вы можете написать мне по адресу [email protected].

CC Saxon Physics Test Forms Руководство по решениям (для семей классических разговоров)

Требуется более подробная информация в ответах на тест Saxon Physics ?

Совместно с издателями Saxon Physics , Classical Conversations® MultiMedia разработала это Test Forms Solutions Manual специально для семей классических разговоров. Он предоставляет пошаговые решения для всех тестов из групп уроков, а также для четверть, полуфиналов и выпускных экзаменов.

Обратите внимание, что Руководство по решениям Test Forms Solutions Manual может быть приобретено только семьями, которые приобрели Saxon Physics Homeschool Kit .

Что такое Saxon Physics Test Forms Solutions Manual?

Physics Test Forms Solutions Manual предлагает расширенные пошаговые решения для тестов, представленных в буклете Saxon’s Test Forms, который включает 25 тестов, квартальные экзамены, семестровые экзамены и заключительный экзамен.Это руководство следует использовать вместе с комплектом Saxon Physics Homeschool Kit. Это издание в мягкой обложке содержит 128 страниц решений и схем.

Этот продукт публикуется Saxon?

No. Classical Conversations® MultiMedia выпустила этот ресурс по эксклюзивному соглашению с издателями Saxon Physics.

Почему был разработан этот продукт?

В своих руководствах по решениям Saxon идет дальше, чем просто дает ответы на набор проблем (ключ ответа). Руководства по решениям показывают каждый шаг проблемы.

Saxon предлагает руководство по решениям для своего учебника Physics , но не для тестов. Продукция Saxon показывает простые ответы на тесты, но не показывает, как получить правильные ответы. Для родителей, плохо знакомых с физикой, это может быть непросто.

Как было разработано руководство по решениям Test Forms Solutions ?

Студенты из Mandala Fellowship прошли все тесты в книге Saxon Physics Test Forms , делая заметки и создавая диаграммы, помимо ответов, данных в Saxon Answer Key .Их ответы были переработаны другими учениками, затем проверены фактами и сравнены с ключом ответа .

Затем результаты были отформатированы так, чтобы они точно соответствовали обозначениям, используемым Saxon в своей учебной программе Physics .

Чем этот продукт отличается от продуктов Physics от Saxon?

Saxon Physics Homeschool Packet включает:

  • Учебник по физике (уроки, наборы задач и ответы на нечетные задачи в приложении)
  • Тестовые бланки (тесты)
  • Answer Key (ответы на тесты)

Saxon также продает:

  • Physics Solutions Manual (пошаговые ответы на наборы задач)

Классические разговоры теперь предлагает:

  • Test Forms Solutions Manual (пошаговые ответы на тесты)

Вы также можете думать об этих продуктах в степени полезности. Вы можете учить, просто используя книгу. Клавиша ответа дает вам еще один инструмент. Руководство по решениям дает вам дополнительную помощь и уверенность.

В ходе производства было обнаружено, что шесть тестовых ответов отличаются от ответов, данных Saxon. Эти конкретные тестовые задачи перечислены во введении, а также отмечены рядом с каждой из них в книге.

Кто может приобрести это Руководство по решениям для тестовых форм

?

Из-за лицензионного соглашения с издателями Saxon, Classical Conversations может продавать эту продукцию только семьям, которые приобрели (или купили) Saxon Physics Homeschool Packet .

.
Leave a Reply

Добавить комментарий

Ваш адрес email не будет опубликован. Обязательные поля помечены *